Você está na página 1de 175

MODUL KERTAS 1 & 2 MODUL 1

4551/1
1

LONJAKAN SAUJANA SPM 2009

Diagram 1 shows a type of muscle tissue found in the human body. Where is the tissue found?

Diagram 1 A B 2 Heart Pancreas C D Biceps Small intestine

The plasma membrane consists of molecules arranged in a double layer as shown in Diagram 2

Diagram 2 The part labelled I and II are A hydrophobic and hydrophilic respectively B hydrophilic and hydrophobic respectively C both hydrophobic D both hydrophilic 3. Diagram 3 shows the net flow of water molecules from a dilute solution to a concentrated solution through a semi-permeable membrane.

Diagram 3 What is this process called? A Osmosis B Active transport C Simple diffusion D Facilitated diffusion

PPD MELAKA TENGAH

4551/1
4

LONJAKAN SAUJANA SPM 2009

Diagram 4 is a graph which shows the changes in mass of potato strips immersed in different concentrations of sucrose solution.

Diagram 4 Based on the graph, which of the following concentrations of sucrose solution should be used so that a flaccid potato strip regains its turgidity? A 1.5 g per 100 ml B 2.5 g per 100 ml C 3.5 g per 100 ml D 4.5 g per 100 ml

5.

Lipid is needed to build substance X while cellulose is needed to build substance Y. What are substances X and Y? Substance X A B Protoplasm Plasma membrane Substance Y Cell wall Cell wall Protoplasm Plasma membrane

C Cell wall D Cell wall

. Haemoglobin is an example of a A. primary structure of protein B. secondary structure of protein C. tertiary structure of protein D. quarternary structure of protein

PPD MELAKA TENGAH

4551/1
7

LONJAKAN SAUJANA SPM 2009

Which of the following graph shows the effect of pH on the metabolic rate of the enzyme pepsin?

Rate of reaction

Rate of reaction

Rate of reaction

Rate of reaction

Diagram 5 shows the phases in a cell cycle.


X phase

X phase

Y phase

Diagram 5

PPD MELAKA TENGAH

4551/1

LONJAKAN SAUJANA SPM 2009

Which of the following represents V ? A B mitosis cytokinesis C D stage S stage G1

Diagram 6 shows a phase in mitosis of a plant cell.

Diagram 6 Which of the following is true about the cell in Diagram 6 ? Stage of mitosis Number of chromosomes in the mother cell 4 4 8 8

A B C D

Anaphase Telophase Anaphase Telophase

10. Crossing over is an important process in meiosis. It results in variations in the daughter cells. At which stage of meiosis does crossing over take place? A B C D Prophase I Prophase II Metaphase II Anaphase I

11

Lack of vitamin D in the diet will cause the disease A B C D scurvy. beri-beri. rickets. pellagra.

PPD MELAKA TENGAH

4551/1
12

LONJAKAN SAUJANA SPM 2009

Diagram 7 shows the molecular structure of three food classes

Diagram 7

Which food classes do X, Y and Z belong to ? X A Carbohydrate B Protein C Protein D Lipid Y Protein Lipid Carbohydrate Carbohydrate Z Lipid Carbohydrate Lipid Protein

13

Diagram 8 shows part of the human alimentary canal.

Diagram 8 A person who has structure P removed must regulate his dietary intake of A. fats B. carbohydrate C protein D. water

14

In the absence of oxygen the skeletal muscles contract using energy from the breakdown of glucose and glycogen to A ethanol and water B energy and water C ethanol and lactic acid D lactic acid and energy

PPD MELAKA TENGAH

4551/1
15

LONJAKAN SAUJANA SPM 2009

Which of the following is not involved in the transportation of carbon dioxide by the blood. A Carbonic acid B Carbaminohaemoglobin C Hydrogen carbonate ion D Carbon monoxide

16

Table shows the biomass of a few types of organisms in a community. Organism Total Biomass of all the organisms at each trophic level(kg) 1200 30 150 670 100 2700

P Q R S T U

Which of the following is a possible food chain in this community? A PUST B QRSP C PSTQ D USPQ

17

Which of the following organisms is a parasite ?

PPD MELAKA TENGAH

4551/1
18

LONJAKAN SAUJANA SPM 2009

Diagram 9 is a graph which shows changes in the population of two species of beetles, T and U, over a period of time. Both species feed on the same food source.

T Population

Diagram 9 What is the interrelationship between species T and U? A Symbiosis B Predation C Parasitism D Competition

19

Diagram 10 shows a cross section of a young dicotyledon root consisting of a few main tissues.

Diagram 10 What is Y ? A Phloem C Cortex

B D

Xylem Cambium

20

Diagram 11 shows some human bones .

Diagram 11

PPD MELAKA TENGAH

4551/1

9
Which bones are part of the axial skeleton ? A P and S B Q and R C P , Q and R D Q , R and S

LONJAKAN SAUJANA SPM 2009

21

Diagram 12 below shows a motor neuron.

Diagram 12

Which one of the following A, B, C or D in the table below names the labelled parts correctly? 1 Cell body Axon Dendrite Synapse 2 Axon Dendrite Myelin sheath Dendrite

A B C D

22

Diagram 13 shows a plant with soft stem . Which of the following support structures helps the plant climb to obtain sunlight ?

Diagram 13

A C

Clasping roots Tendrils

B D

Twining stems Thorns.

PPD MELAKA TENGAH

4551/1
23

10

LONJAKAN SAUJANA SPM 2009

Diagram 14 shows a nerve pathway involved in a reflex action. Which structure is the efferent neurone?

Diagram 14

24

Some cucumber slices are immersed in 0.1% sucrose solution. After 3 hours, the slices are found to be turgid and hard. Which of the following statements explains this phenomenon? A B C D The cucumber cell wall prevents it from shrinking The cell sap is hypotonic towards the sucrose solution The high concentration of the cell sap in the vacuole causes water to diffuse into the cell The cucumber cell wall allows the sucrose molecules to diffuse into the cell

25

Diagram 15 shows the four-chambered stomach of a ruminant.

Diagram 15 Which of the following is not a correct match about each chamber and its function? A. S- Rumen, mutualistic bacteria digest cellulose. B. V- Reticulum, the partially digested food is formed into balls and regurgitated into the mouth for chewing C. U- Omasum, the regurgitated food in the mouth is passed into the omasum D. T- Duodenum, the digested products are absorbed.

PPD MELAKA TENGAH

4551/1
26

11

LONJAKAN SAUJANA SPM 2009

What substances are dissolved in the fuid which passes along the ureter to the bladder of a healthy person? Glucose Absent Absent Present Present Protein Absent Absent Absent Present Salts Absent Present Present Absent Urea Present Present Present Absent

A. B. C. D.

27

Diagram 16 shows the female reproductive system. In which parts are the eggs and the zygote formed? 1 2

Diagram 16 Eggs A B C D 1 1 2 2 Zygote 2 3 1 3

28

Diagram 17 shows sex determination in human.

Ovary cell

Testis cell

P R
Diagram 17

Gametes

PPD MELAKA TENGAH

4551/1

12

LONJAKAN SAUJANA SPM 2009

If a couple have a son, what is the cell composition in P, Q and R? P 44 + X 22 + Y 22 + X 22 + X Q 44 + Y 22 + X 22 + X 22 + Y R 44 + XY 44 + XY 44 + XX 44 + XY

A B C D 29

Which of these genetically inherited disease is dangerous and can cause death at a young age? A. B. C. D. Albino Haemophilia Short-sightedness Down syndrome

30

Table 1 shows a Punnet square which represents the gametes and progeny from a dihybrid cross. Alphabets a to p represent the daughter cells from this cross. male gamete female gamete HK Hk hK hk HK Hk hK hk

a e I m Table 1

b f j n

c g k o

d h l p

Which of the following daughter cells have the same genotype as the parent ? A B a, f, k, p b, c, e, I C D d, g, j, m e, f, h, l

31

Diagram 18 shows the development of a pollen tube and its entry into the ovule. Which part develops into the testa after fertilisation?

Diagram 18

PPD MELAKA TENGAH

4551/1

13

LONJAKAN SAUJANA SPM 2009

32

Contraceptive pills contain a combination of A. estrogen and luteinising hormone. B. progesterone and prolactin C. estrogen and follicle stimulating hormone D. progesterone and estrogen.

33. Which of the following shows the differences between mitosis and meiosis? Mitosis Involves one stage of cell division Produces two diploid daughter cells Synapsis and crossing over takes place between homologous chromosomes Chromosomes are not in pairs Meiosis Involves two stages of cell division Produces four haploid daughter cells Synapsis and crossing over does not take place Homologous chromosomes are in pairs at prophase I

I II III

IV

A B C D 34

I and II only I and III only I, II and IV only I, II, III and IV Diagram 19 is a graph which represents a type of variation found in students.

Number of individuals

Variation
Diagram 19 This variation may be I III A B height type of ear lobe I and II only I and III only II weight IV dimple III and IV only I, II, III and IV

C D

PPD MELAKA TENGAH

4551/1
35

14
Which of the following is not a cause of variation? A. C. Radiation Asexual reproduction B. D.

LONJAKAN SAUJANA SPM 2009

Gene mutation Meiosis

36

Diagram 20 shows the structure of a villus in the ileum

Diagram 20 Which vessels P, Q, R, and S carry the largest amounts of glucose, amino acids, lipid droplets or fat-soluble vitamins?

Glucose A B C D P Q R S

Amino acids Q P R S

Lipid droplets R R S R

Fat soluble vitamins S S S R

37

Diagram 21 shows a plastic quadrat used to determine the percentage coverage of bread mould on a piece of bread. The shaded area shows the presence of mould.

Diagram 21

Based on the result obtained in Diagram 21, calculate the percentage coverage of the bread mould , taking the area of each small square to be 1 cm A 32 % B 40% C 56% D 80%

PPD MELAKA TENGAH

4551/1

15

LONJAKAN SAUJANA SPM 2009

38

Diagram 22 below shows equipment that can cause the thinning of the ozone layer.

Diagram 22 How can this problem be solved? I II III IV A B C D Stop using chlorofluorocarbon Replace CFCs with HCFCs Patching holes in the ozone layer Produce less electrical goods I and II II and IV I, II and III I, II, III and IV

39

The following statement is about eutrophication. L : M: N : O : Excess fertilisers from agriculture lands flow into lakes Bacteria grow rapidly Algae grow rapidly and covers the surface of the lake The value of BOD increase

What is the correct sequence of the eutrophication process ? A O, L, M and N B L, N, M and O C L, M, O and N D O, M, L and N

PPD MELAKA TENGAH

4551/1
40.

16

LONJAKAN SAUJANA SPM 2009

An experiment was carried out to investigate the rate of water loss from a plant in a day.The wind and relative humidity factors were kept constant. Diagram 23 is a graph which shows the result obtained from 0600 to 1300 hours .Which of the curves A,B,C or D is expected to show the rate of water loss in the plant after 1300 hours ?

Diagram 23 41
Diagram 24 shows a human arm .

Diagram 24 If tendon X was torn off,what happens to the arm ? A The elbow joint loosens up B The fingers cannot grip C The arm connot be bent D The lower arm cannot twist.

PPD MELAKA TENGAH

4551/1
42

17

LONJAKAN SAUJANA SPM 2009

Diagram 25 is a graph which shows the changes in the glucose concentration in the blood of a person over a period of two hours.

Diagram 25 Which of the following best explains the shape of the graph after X? A. The person has eaten a meal that is high in sugar . B. The person has had an insulin injection. C. The person is suffering from diabetes mellitus. D. The person starts some vigorous physical exercise.

43

Which characteristics of the glomerulus enhances the efficiency of ultrafiltration ? I. The diameter of the afferent arteriole is larger than that of the efferent arteriole. II. The afferent arteriole divides further into a dense network of capillaries. III. The high hydrostatic pressure of the blood entering the glomerulus. IV. The Bowmans capsule is made up of only two layers of cells. A. B. C. D. I, II and III only I, II and IV only II, III and IV only I, II, III and IV

44

A womans menstrual period started on 23rd March. In which week was an egg most likely to have been released?

Week A B C D Sun 5 12 19 26 Mon 6 13 20 27 Tue 7 14 21 28

March Wed 1 8 15 22 29

Thu 2 9 16 23 30

Fri 3 10 17 24 31

Sat 4 11 18 25

PPD MELAKA TENGAH

4551/1
45

18

LONJAKAN SAUJANA SPM 2009

What is true about the importance of secondary growth in plants? I. It increases the diameters of the plant stems and roots for mechanical support. II. It allows plants to increase in length to achieve maximum height. III. It produces new phloem and xylem tissues to replace the old and damaged ones. IV. It produces a thick bark which reduces the evaporation of water from the surface of the stem.

A. I, II and III only C II, III and IV only

B. D.

I, III and IV only I, II, III and IV.

46

A woman with blood group A claims that a man with blood group AB is the father of her baby. The babys blood is tested. Which of the following could be the babys blood group? I II III IV A B C D Group A Group B Group O Group AB I and II only I and IV only I, II and IV only I, II, III and IV

47

Which of the following shows the difference between continuous variation and discontinuous variation?

A B C D

Continuous Variation Controlled by dominant genes. Caused by mutation. Occurs in animals. Can be measured.

Discontinuous Variation Controlled by recessive genes. Not caused by mutation. Occurs in plants. Cannot be measured.

48

The Hydrangea plant produces blue flowers when grown on acidic soil, and red flowers when grown on alkaline soil. What conclusion can be made from this observation? A. B. C. D. The colour of the Hydrangea flower is a continuous variation The environment affects the colour of the flowers The pH of the soil causes mutation The colour of the flower is affected by the genetic factor only

PPD MELAKA TENGAH

4551/1
49

19

LONJAKAN SAUJANA SPM 2009

I - Homologous chromosomes line up at the equator of the cell. II - Homologous chromosomes separate and move to opposite poles III Sister chromatids separate and move to different poles IV Nuclear membrane and nucleolus disintegrate. I, II, III and IV shows the processes which occur in meiosis. Among the following events, which occur in anaphase I ?. A B C D I only II only III and IV only II and IV only

50

Diagram 26 shows an organ system.

Diagram 26

What are the functions of the organ system shown above? I To transport oxygen to the body cells II To defend the body against diseases III To remove metabolic wastes IV To help regulate the volume and composition of blood A I and II only B I and III only C II and IV only D III and IV only

END OF QUESTION PAPER KERTAS SOALAN TAMAT

PPD MELAKA TENGAH

4551/2

-2-

LONJAKAN SAUJANA SPM 2009

Section A [ 60 marks ] Answer all questions from this section. 1. Diagram 1 shows cell organisation in plant. Cells J undergo differentiation and specialisation to form several tissues in a leaf of a green plant. Rajah 1 menunjukkan organisasi sel dalam satu tumbuhan. Sel-sel J mengalami pembezaan dan pengkhususan untuk membentuk beberapa tisu dalamsatu tumbuhan hijau.

Cells J Sel-sel J

Cell Specialisation

L Cross-section of a leaf Keratan rentas sehelai daun

Xylem tissue Tisu xilem

M DIAGRAM 1

(a)

Name tissue K and tissue L. Namakan tisu K dan L K : L: [2 marks]

PPD MELAKA TENGAH

4551/2
(b)

-3-

LONJAKAN SAUJANA SPM 2009

State the function of cells K and M in a leaf. Nyatakan fungsi sel K dan M dalam sehelai daun

K : . M: ... [2 marks] (c) (i) Explain the differentiation of cells J to form the xylem tissue. Terangkan pembezaan sel J dalam membentuk tisu xilem. [2 marks] (ii) During the formation of the xylem tissue, the plant was unable to synthesise lignin. Explain the effect on the function of the leaf. Sewaktu pembentukan tisu xylem, satu tumbuhan gagal mensistesis lignin. Terangkan kesannya keatas fungsi daun tumbuhan tersebut. [2 marks] (d) Based on diagram 1, state the meaning of cell specialization. Berdasarkan rajah 1, nyatakan maksud pengkhususan sel.

[2 marks] (e) Leaf is the main photosynthetic organ of a plant. Explain the adaptation of tissue L to enable the leaf to carry out its function. Daun adalah organ utama fotosintesis sesuatu tumbuhan Terangkan adaptasi tisu L untuk membolehkannya menjalankan fungsinya

TOTAL [2 marks] PPD MELAKA TENGAH

4551/2
2.

-4-

LONJAKAN SAUJANA SPM 2009

Diagram 2.1 shows the cell cycle of an organism.

Mitotic cell division

Phase U

Diagram 2.1 (a) . Name phase U in Diagram 2.1. U: [1 mark] (b) Phase U is further divided into three sub phases, X, Y and Z. Describe what happens at sub phases X, Y and Z. X: Y: Z: [3 marks] (c) The number of chromosomes present in the nucleus of a somatic cell is 6. Diagram 2.2 shows a stage of cell division to produce gametes.

Diagram 2.2

PPD MELAKA TENGAH

4551/2

-5-

LONJAKAN SAUJANA SPM 2009

Complete the diagram to show the chromosomes for a daughter cell produced at the end of sub-phase Q.

[2 marks]

(d)

Explain how radiotherapy affected cell cycle in cancer treatment. [2 marks]

(e)(i)

A farmer wants to breed a good variety of banana plants for commercial production. Suggest a suitable method to be used and explain how the method named can increase the crop yield. [3 marks]

(ii)

State a problem that can occur when using this method.. [1 mark] TOTAL

PPD MELAKA TENGAH

4551/2
3.

-6Diagram 3 shows part of a nitrogen cycle. Rajah 3 menunjukkan sebahagian kitar nitrogen. Nitrogen in the atmosphere DIAGRAM 4 Nitrogen dalam atmosfera

LONJAKAN SAUJANA SPM 2009

Nitrogen fixation by microorganisms in plant P Pengikatan nitrogen oleh mikroorganisma dalam tumbuhan P

Organism R Organisma R

Substance Q Bahan Q
Nitrogenous compounds in plants Sebatian nitrogen dalam tumbuhan

Nitrites Nitrit

Process Y Proses Y
Ammonium compounds Sebatian ammonium Nitrogenous compounds in animals Sebatian nitrogen dalam haiwan DIAGRAM 3 a) Name P, Q and R Namakan P, Q dan R. P: ......................................................................................................................... Q: ........................................................................................................................ R: .........................................................................................................................

[3marks]

PPD MELAKA TENGAH

4551/2

-7-

LONJAKAN SAUJANA SPM 2009

(b) (i ) Name the microorganism that is involved in the nitrogen cycle and lives in plant P. Namakan mikroorganisma yang terlibat dalam kitar nitrogen dan tinggal dalam tumbuhan P.

.......................................................................................................................................... [ 1 mark]

(ii) Besides nitrogen fixation by microorganisms, name a natural phenomenon which is also able to convert nitrogen in the atmosphere to substance Q. Selain daripada pengikatan nitrogen oleh mikroorganisma, namakan satu fenomena semulajadi yang boleh menukarkan nitrogen dalam atmosfera kepada bahan Q. .......................................................................................................................................... [ 1 mark] (c) Microorganisms are involved in process Y. Mikroorganisma terlibat dalam proses Y. (i) Name one type of microorganism which is involved in process Y. Namakan sejenis mikroorganisma yang terlibat dalam proses Y.

.......................................................................................................................................... [ 1 mark] (ii) Explain the role of the microorganism in ( c)(i) Terangkan peranan mikroorganisma dalam (c)(i)

.......................................................................................................................................... .......................................................................................................................................... .......................................................................................................................................... .......................................................................................................................................... [ 3 marks] (d) Explain how a deficiency of Q in the soil affects the growth of the plants. Terangkan bagaimana kekurangan Q dalam tanah memberi kesan terhadap pertumbuhan tumbuhan. .......................................................................................................................................... .......................................................................................................................................... .......................................................................................................................................... [ 3 marks] PPD MELAKA TENGAH TOTAL

4551/2

-8-

LONJAKAN SAUJANA SPM 2009

Two individuals P and Q were given injections to acquire immunity. The level of antibodies in the blood of individual P and Q is shown in Diagram 4.1 and 4.2 respectively. Individual P
Booster dose (2nd) stimulates a faster and larger lasting response.
Immunity level

Concentration of antibodies in the blood (arbitrary unit)

0
st

3
nd

st 1 1vaccination Injection

2 vaccination 2nd Injection

Time (weeks)

DIAGRAM 4.1

Individual Q
Increase immediately
Concentration of antibodies in the blood (arbitrary unit)

Immunity level

Time (weeks)

3
2
nd

1st injection

injection DIAGRAM 5.2

DIAGRAM 4.2 (a) What is the substance injected into the blood of individual P and individual Q ? P : Q : PPD MELAKA TENGAH

4551/2

-9-

LONJAKAN SAUJANA SPM 2009 [ 2 marks ]

(b) State the type of immunity obtained by individual P and individual Q. P : .. Q : .. [ 2 marks ] A boy was bitten by a snake. He was unconscious and he was hospitalised.

(c) Using your biological knowledge, describe how you could save this boy. ... [ 4 marks ] (d) Table 4.1 shows a schedule of immunisation given for every new born Malaysian until the age of two. Age New born 1 month 3 month 5 month Types of Immunity Tuberculosis (B.C.G) Hepatitis B Hepatitis B Triple Antigen Polio Triple Antigen Polio Hepatitis B 9 24 month 1 - 2 year Germans measles Triple Antigen Polio ( Third dose ) TABLE 4.1 ( First dose ) ( Second dose ) ( First dose ) ( Second dose ) ( Third dose )

PPD MELAKA TENGAH

4551/2
2009

- 10 -

LONJAKAN SAUJANA SPM

(d) (i) Based on Table 4.1, state the type of pathogen which cause the diseases. ................................................................................................................................................... [1 mark] (ii) Explain why there is a need for second and third doses for the immunisation. [ 3 marks ] TOTAL

PPD MELAKA TENGAH

4551/2

- 11 -

LONJAKAN SAUJANA SPM 2009

5.

Figure 5 shows various types of fingerprints. Rajah 5 menunjukkan bebrapa jenis cap jari

Composite Komposit

Whorl Pusar FIGURE 5

Curves Lengkung

Loops Gelung

(a) (i)

Based on Figure 5, name the type of fingerprints of students X and Y below. Berdasarkan Rajah 5, namakan jenis cap jari bagi pelajar X dan Y di bawah. Student X Student Y

Type of fingerprint: Jenis cap ibu jari:

Type of fingerprint: Jenis cap ibu jari:

. [2 marks]

(ii) State one factor that causes variation in the fingerprints of students X and Y. Nyatakan bagaimana faktor di (a)(ii) menghasilkan variasi. .... .. [1 mark] (iii) State how the factor in (a) (ii) causes variation. Nyatakan bagaimana faktor di (a)(ii) menghasilkan variasi. [1 mark] PPD MELAKA TENGAH

4551/2
(b)

- 12 -

LONJAKAN SAUJANA SPM 2009

(i) What is the type of variation shown in Figure 5? Apakah jenis variasi yang ditunjukkan dalam Rajah 5? [1 mark]

(ii) State two traits, other than fingerprint, which show the same type of variation as in (b)(i). Nyatakan dua trait selain cap jari yang menunjukkan variasi yang sama seperti (b)(i). Trait 1 : Trait 2 : [2 marks]

(c)

Height is a type of variation. Explain the differences between the type of variation shown by fingerprints and height. Trait ketinggian merupakan sejenis variasi Huraikan perbezaan antara variasi yang ditunjukkan oleh trait jenis cap jari dengan trait ketinggian pelajar. [2 marks]

(d)

Explain how variation can ensure the survival of a species. Terangkan bagaimana variasi boleh menjamin kemandirian suatu spesies [3 marks] TOTAL

PPD MELAKA TENGAH

4551/2

- 13 -

LONJAKAN SAUJANA SPM 2009

SECTION B [ 40 marks ] Answer two questions from this section.

6.

Diagram 6.1 shows a respiratory structure of an insect. Rajah 6.1 menunjukkan struktur respirasi satu serangga P Tracheol Trakeol

Body cells Sel-sel Badan DIAGRAM 6.1

(a) (i) Explain the gases exchange between tracheol and body cell. Terangkan pertukaran gas antara trakeol dan sel-sel badan [4 marks]

(ii) Chitin is a polysaccharide on the outer surface of structure P. Due to the change in the environment, the insect is unable to form the polysaccharide. Explain how the absence of chitin affects inhalation and the energy production. Kitin adalah polisakarida yang terdapat pada permukaan struktur P. Disebabkan perubahan dalam persekitaran, serangga tidak dapat menghasilkan polisakarida. Terangkan bagaimana ketiadaan kitin memberi kesan keatas proses tarikan nafas dan penghasilan tenaga . [6 marks]

PPD MELAKA TENGAH

4551/2

- 14 -

LONJAKAN SAUJANA SPM 2009

(b) Diagram 6.2 shows the rate of oxygen intake before, during and after a vigorous exercise of an athlete. Rajah 6.2 menunjukkan kadar pengambilan oksigen sebelum, semasa dan selepas satu latihan intensif seorang atlit. Oxygen intake (litre/minute)

Vigorous exercise DIAGRAM 6.2

Time (min)

(i) Based on the graph, compare the respiration before and during the vigorous exercise. Berdasarkan graf diatas, bezakan proses respirasi sebelum dan semasa latihan tersebut. [4 marks]

(ii) Explain how the oxygen intake by the athlete returns to the normal level at the 25th minute. Terangkan bagaiman selepas minit ke 25 pengambilan oksigen oleh atlit tersebut kembali ke asal [6 marks]

PPD MELAKA TENGAH

4551/2

- 15 -

LONJAKAN SAUJANA SPM 2009

(7)

(a) Figure 7 shows development of the follicle in the female ovary, thickening of uterine endometrium and the hormones involved. Rajah 7 menunjukkan perkembangan folokel dalam ovari seorang perempuan, penebalan endometrium uterus dan hormonphormon yang terlbat.

Estrogen

Time (Day)
FIGURE 7 Explain the relationship between development of the follicle , changing of the respective hormonal level in the blood and the thickening of the uterine endometrium in a female. Terangkan hubungan antara perkembangan folikel, perubahan aras hormonhormon masing-masing dalam darah dan penebalan endometrium uterus pada seorang perempuan. (10marks)

PPD MELAKA TENGAH

4551/2

- 16 -

LONJAKAN SAUJANA SPM 2009

(b) Graph 7(a) and 7(b) show the growth curve of human and insect. Based on the graph , compare the growth process in human and insect. Graf 7 (a) dan 7 (b) menunjkkan lengkuk pertumbuhan manusia dan serangga Berdasarkan graf, bandingkan proses pertumbuhan pada manusia dan serangga. (10marks)

Heigh t (cm)

Time (year) GRAPH 7(a) : Growth curve for human

Length (cm)

Time(day) GRAPH 7(b) : Growth curve for insect

PPD MELAKA TENGAH

4551/2
8 (a)

- 17 -

LONJAKAN SAUJANA SPM 2009

Diagram 8 shows the blood groups of a married couple, Encik Ahmad and Puan Amalina and their children. Rajah 8 menunjukkan kumpulan darah bagi pasangan suami isteri Encik Ahmad dan Puan Amalina serta anak-anaknya.

Parents Ibu bapa

E En Ahmad Blood group A Kumpulan darah A darahBloogroup A 0ffspring Anak

EPn Amalina Blood group B Kumpulan darah B Blood group A

blood group 0 kumpulan darah O

blood group 0 kumpulan darah O

blood group 0 blood group AB kumpulan darah O kumpulann darah AB

Diagram 8 Diagram 8 shows the variation of blood groups in En Ahmads family. Explain why there is a variation in blood groups of the offspring. Rajah 8 menunjukkan variasi kumpulan darah dalam keluarga En Ahmad. Terangkan mengapa adanya variasi dalam kumpulan darah anak-anaknya. [ 10 marks] (b) Genetic engineering is widely used in the field of agriculture and medicine. Justify the impact of genetic engineering on humans and the environment. Kejuruteraan genetik digunakan secara meluas dalam bidang pertanian dan perubatan. Beri wajaran tentang impak kejuruteraan genetik terhadap manusia dan persekitaran. [ 10 marks]

PPD MELAKA TENGAH

4551/2

- 18 -

LONJAKAN SAUJANA SPM 2009

Biodiversity is the variety of plants, animals and microorganisms living on Earth. These organisms live in different ecosystems and are important to our lives. Biodiversiti ialah kepelbagaian jenis tumbuhan, haiwan dan Mikroorganisma yang hidup di bumi. Organisma ini hidup dalam berbagai ekosistem dan penting kepada kehidupan kita. 9.(a)(i) Based on the statement discuss the importance of biodiversity. Berdasarkan pernyataan di atas bincangkan kepentingan biodiversiti. [4 marks] (ii) Diagram 9 shows an ecosystem in Malaysia. Rajah 9 menunjukkan satu ekosistem di Malaysia

Diagram 9 Discuss the importance of the ecosystem shown in Diagram 8 to the environment and economy of our country. Bincangkan kepentingan ekosistem di Rajah 8 kepada persekitaran dan ekonomi negara kita. [6 marks] (b) Biotechnology is the application of organisms or microorganisms or their biological processes in the production of materials for use in medicine and industry. Biotechnology ialah aplikasi organisma atau microorganism atau proses biologi dalam penghasilan bahan-bahan untuk kegunaan bidang perubatan dan perindustrian. Discuss the uses of microorganisms in Bincangkan pengunaan microorganisma dalam (i) the waste treatment process. proses rawatan kumbahan. food processing pemprosesan makanan [10 marks] END OF QUESTION PAPER PPD MELAKA TENGAH

(ii)

PPD MELAKA TENGAH LONJAKAN SAUJANA SPM 2009 JAWAPAN - BIOLOGY PAPER 1

1. A 2. B 3. A 4. A 5. B 6. D 7. C 8. B 9. A 10. A

11. C 12. C 13. A

21. C 22. C 23. C 24. C 25. D 26. B 27. C 28. D 29. B 30. C

31. C 32. D 33. D 34. C 35. C 36. C 37. C 38. C 39. B 40. D

41. C 42. A 43. A 44. B 45. B 46. C 47. D 48. B 49. B 50. D

14. D 15. D 16. C 17. B 18. D 19. A 20. C

PPDMT LONJAKAN SAUJANA SPM 2009

BIOLOGY P2

MARKING SCHEME - BIOLOGY PAPER 2


QUESTION 1 No (a) Criteria Able to name tissue K and tissue L. Answer: K: Upper epidermis (cells / tissue) L: Palisade mesophyll (cells / tissue) Able to state the function of cells K and M in a leaf. Sample answer: K: Protect the inner tissues. // Allows light to penetrate. M: Controls the size of stoma / transpiration / gaseous exchange // Allows gaseous exchange through the stoma. Able to explain the differentiation of cells J to form the xylem tissue. Sample answer: Cells J join end to end, / the wall of cells J at the joints dissolved, to form a hollow tube / continuous tube (from root to leaves). The wall of xylem vessel is thickened by lignin. (Any 2) Able to explain the effect on the function of the leaf when the plant unable to synthesise lignin during the formation of the xylem tissue. Sample answer: Xylem cannot be strengthened / cannot uphold leaf. Less sunlight received / absorbed. Slow down the rate of photosynthesis / less glucose produced Or (Any 2) Xylem vessels collapsed. Less water supplied to leaves. Slow down the rate of photosynthesis / less glucose produced (Any 2) Able to state the meaning of cell specialisation. Sample answer: Cells grow, change shape / differentiate. To carry out / perform specific function. Able to explain the adaptation of palisade mesophyll tissue to enable the leaf to carry out its function. Sample answer: Upright and closely packed. Contains large number of chloroplast. All cells receive maximum amount of sunlight. // Absorb maximum amount of sunlight // energy. TOTAL Marks

1 1

(b)

1 1 2

(c) (i)

1 1 1

(ii)

1 1 1

1 1 1

(d)

1 1

(e)

1 1 1 2 12

PPDMT LONJAKAN SAUJANA SPM 2009

BIOLOGY P2

QUESTION 2 No Criteria (a) Able to name the phase U. Sample answer: U : Interphase (b) Able to describe the processes at sub phases X, Y and Z during phase U Sample answer : X : Cell synthesises protein / new orgenelles formed Y : DNA is synthesized / is replicated / 2 sister chromatids formed Z : Cell accumulates energy / synthesise energy / prepare for cell division (c) Able to draw a daughter cell based on the following criteria: No. of chromosomes are haploid / 3 chromosomes Types of chromosomes/ non homologous New genetic combination (d) Able to explain how radiotherapy can treat cancer. Sample answer : F : Radiotherapy uses radiation / high energy rays E1 : destroy the nucleus of cancerous cells E2 : cancerous cells die / cannot divide mitotically E3 : cell cycle stops (e)(i) Able to name the method and explain the advantages of the method in increasing crop yield. Sample answer : T : Tissue culture / Cloning E1 : Large numbers of clones can be produced E2 : Within a short period of time / any time E3 : Clones inherited good characteristics/ resistance to diseases / fast growth rate / large fruit / good genetic traits (e)(ii) Able to state one problem : Clones can be destroyed completely if they do not have the resistance to new diseases / pest.// No variation

Marks

1 1 1 3

1 1 1 Any 2

1 1 1 1 Any 1E = 1

1 1 1 1 T=1m Any 2E Marks

Any 1

TOTAL

12

PPDMT LONJAKAN SAUJANA SPM 2009

BIOLOGY P2

QUESTION 3 No Criteria Marks

3a

Able to name P,Q and R Suggested answer P: leguminous plant / example of a leguminous plant Q: nitrates R: denitrifying bacteria Able to state the name of the microorganism Suggested answer Rhizobium sp /nitrogen fixing bacteria Able to name the natural phenomenon that can convert atmospheric nitrogen to substance Q Suggested answer Lightning Able to suggest the type of the microorganism that is involved in process Y. Suggested answer saprophytic bacteria / fungi // putrefying bacteria / fungi Able to explain the role of the microorganism in the nitrogen cycle Suggested answer 1. Saprophytic bacteria / fungi decompose protein in the dead plants and animals / excretory products of animals 2. to ammonium/ simpler nitrogenous compounds/ ammonia which is eventually converted to nitrates. 3. This increases the nitrate / nitrogen content of the soil. Able to explain how a deficiency of Q in the soil affects plant growth. Suggested answer 1 Root hairs absorb less Q/nitrates/nitrogen // less Q/nitrates/nitrogen is available to be absorbed by plants. 2. Less chlorophyll / protein is synthesized. 3 Rate of photosynthesis decreases. 4 Plant growth is slow / retarded. Any 3

1 1 1

b(i)

(ii)

c(i)

(ii)

1 1 1 3

1 1 1 1

TOTAL

12

PPDMT LONJAKAN SAUJANA SPM 2009

BIOLOGY P2

QUESTION 4 No Criteria (a) Able to state the substances injected into the blood of individual P and individual Q. Sample answer: P : Dead or weakened bacteria / viruses / antigens// vaccine Q : Serum containing antibodies // antiserum (b) Able to explain the type of immunity obtained by individual P and individual Q. Sample answer : P : Artificial active immunity The body produces its own antibodies to fight against infections by pathogens. Q : Artificial passive immunity The body receives antibodies produced from outside sources to fight against infections by pathogens. Able to describe how could save that boy. Sample answer : F1: Snake venom / toxin acts as antigen to our body F2: Injection of serum which contains instant antibodies / antiserum / anti-toxin must be given to the patient. F3: Antibody-antigen action occured very fast F4: Antitoxin/ antibody reacts with toxin / snake venom/ antigen and neutralize it / he is saved. Able to state the types of pathogen which cause the diseases Answer: Virus / bacteria Able to explain why there is a need for second and third doses for the immunisation Sample answer : F1: Immunisation is given to prevent infection from pathogens that caused diseases like Tuberculosis, Hepatitis B, Polio, diphtheria, whooping cough, tetanus. German measles ( state at least 2 example ) F2: New born are injected with vaccines to get Artificially Active Immunity F3: First dose are given to induce baby lymphocytes to produce antibodies which are specific against the antigens / bacteria / virus F4: 2nd and 3rd dose are booster dose to increase the production of antibodies at a faster rate. F5: Achieved immunity level // antibodies remained in the blood for a long time and provide permanent immunity / protect them from the next infection. Any 3

Marks

1 1

(c)

1 1 4 1 1

(d)(i)

(ii)

1 1

TOTAL

12

PPDMT LONJAKAN SAUJANA SPM 2009

BIOLOGY P2

QUESTION 5 No (a) (i) Criteria Able to name the type of fingerprints of students X and Y Answer: X - Loop ; Y- Composite Able to state one factor that causes variation in the fingerprints of students X and Y. Answer: Genetic factor Able to state how the factor in (a) (ii) causes variation Answer: Genetic recombination during crossing over results in the formation of different Able to state the type of variation Answer: Continuos variation Able to state two traits, other than fingerprint, which show the same type of variation as in (b)(i) Answer: The ability to roll tongue Types of hair Able to explain the differences between the type of variation shown by fingerprints and height. Sample answer: Height Shows normal distribution Affected by environmental Factors Types of fingerprint Shows discrete distribution Not affected by environmental factor 1 1 2 1 1 2 Marks

( ii)

(iii)

(b) (i)

(ii)

(c)

Able to explain how variation can ensure the survival of a species (d) Sample answer: - Can differentiate from one individual to another / no one is the same - Able to adapt to a new environment - Able to camourflage to run away from any predators 1 1 1 3

TOTAL

12

PPDMT LONJAKAN SAUJANA SPM 2009

BIOLOGY P2

SECTION B QUESTION 6 No (a) (i) Criteria Able to explain the exchange of gases between tracheole and body cell. Sample answer: Partial pressure/concentration of oxygen in the tracheole is higher than partial pressure/concentration of oxygen in body cell . Oxygen diffuse from tracheole to body cell Partial pressure/concentration of carbon dioxide in the body cell is higher than partial pressure/concentration of carbon dioxide in tracheole . Carbon dioxide diffuse from tracheole to body cell Able to explain how the absent of chitin affect the process of inhalation and energy production of the insect. Sample answer: The function of chitin is to prevent trachea from collapsing/sustain the air pressure During inhalation high pressure air moves into the trachea. The absent of chitin will cause the trachea / P to collapse / burst / rupture. Air with oxygen cannot reach tracheal. Body cell cannot get enough oxygen for cellular respiration The insect does not produce enough energy and respire anaerobically. Less energy produced. (Any 6) b (i) Able to compare and explain the respiration before and during vigorous exercise. Sample answer: Before (A) Aerobic Respiration During (B) Anaerobic Respiration Explanation (E) Before - oxygen intake is low/the same as oxygen required/enough oxygen is supplied to the cell During oxygen required is more than oxygen intake Before oxygen is sufficient During oxygen is insufficient / oxygen supplied is less than oxygen supplied. Before complete break down of glucose (produce Marks

1 1 1 1 4

(ii)

1 1 1 1 1 1 6 1

1 .

2 .

The muscles are in normal condition

The muscles are in the state of oxygen debt

3 .

Energy produced is

Energy produced is

PPDMT LONJAKAN SAUJANA SPM 2009

BIOLOGY P2

more/38 ATP

less / 2 ATP

more energy) During incomplete break down of glucose (produce less energy) Before complete break down of glucose produce carbon dioxide and water During Incomplete breakdown of glucose produce lactic acid A + B = 1m E=1m (Any one E) 8

4 .

No/less accumulatio n of lactic acid in the muscles

High accumulation of lactic acid in the muscles

(b) (ii)

Able to explain how the oxygen intake by the athlete returns to the normal level at the 25th minute. Sample answer: Lactic acid has been removed from the muscles The lactic acid has been converted to energy/ convert to glucose TOTAL

1 1

2 20

PPDMT LONJAKAN SAUJANA SPM 2009

BIOLOGY P2

QUESTION 7 No. 7(a) Criteria Able to explain the relationship between development of the follicle , changing of the respective hormonal level in the blood and the thickening of the uterine endometrium. Suggested answer: Day 0 7 Follicle very small start to develop when receive FSH from pituitary the wall of follicle will produce estrogen FSH - pituitary start to release FSH, FSH will go to the ovary - FSH stimulate development of follicle Endometrium - stimulate by estrogen; undergo thickening / repairing Day 8 14 Follicle - become larger, develop to form follicle Graaf FSH / LH/ Estrogen/progesterone - FSH decrease, LH at maximum level , estrogen at maximum level - LH stimulate ovulation / completion of meiosis I, estrogen stimulate the thickening of endometrium - Progesterone level very low Endometrium - endometrium become very thick (ready to implantation (of embryo)) Day 15 - 21 Follicle - Follicle undergoes ovulation/ released oocyte II - The remaining follicle tissue / corpus luteum secreted small amount of estrogen but large amount of progesterone FSH/ LH/ Estrogen/ Progesterone - Progesterone stimulated the thickening of endometrium , halted the secretion of FSH/LH - Development of new follicle and ovulation stop. Endometrium - more thicker and highly vascular - ready for implantation of embryo Mark Remark

1 1 1 1 1

Max 3 marks for each stage - 9 marks

1 1 1 1

At least the answer shows the relationship between 3 parameter i.e follicle, hormone and endometriu m - 1 mark

1 1

1 1

PPDMT LONJAKAN SAUJANA SPM 2009

BIOLOGY P2

Day 22 28 Corpus luteum - if no fertilisation, corpus luteum become disintegrate FSH/ LH/ Estrogen / Progesterone - FSH, LH and estrogen at minimum level; progesterone level also drop Endometrium - endometrium become breakdown & disintegrate - blood and tissue are shed / lining of uterus discharge through vagina as menstrual flow. 7(b) Able to compare the growth process in human and insect. Suggested answers: Similarities - height of man / length of instar increases by time - both show horizontal line / constant growth during adult Difference - Form of graph Sigmoid form for human and like series of steps in insect - Age of organism the height measured yearly, but in insect used day for measuring the length - Caused of different human have endoskeleton but insect have exoskeleton - Stages involve in human, the curve has three different phases, but there are five steps in insect // nymphal stages - Vertical and horizontal line : curve for human did not shows different line (only the curve from continuous points), but there are five different horizontal and vertical lines each - Zero growth no point to show zero growth in human, but there are 5 time of zero growth (at horizontal line) - Sudden growth : no sudden growth for human, but there are sudden growth in insect (at vertical line) - Ecdysis : no ecdysis in human but ecdysis occurred in insect - Mitosis : the cells in human undergo mitosis all the time, but in insect, mitosis only occurred at certain time (during ecdysis) - Absorption of air : in human, there are no absorption of air, but in insect, during ecdysis TOTAL

1 1 Max 10

1 1

Max 10 marks *2 marks for similarities, 8 marks for differences

1 1 1 1

1 1

1 20

10

PPDMT LONJAKAN SAUJANA SPM 2009

BIOLOGY P2

QUESTION 8 No 8(a) Criteria Able to explain why there is a variation of blood groups in the offspring 1. The ABO blood group in humans is controlled by three alleles IA, , IB and Io. 2 Alleles IA and IB are codominant but allele Io is recessive. 3 Ahmad is heterozygous for blood group A // Genotype of Ahmad is IA, IO, 4 Amalina is heterozygous for blood group B // Genotype of Amalina is IB, IO 5 Ahmad produces two types of sperms, one containing allele IA and the other containing allele IO . 6 Amalina produces ovum containing allele IO or allele IB 7 When the sperm containing allele IO fertilizes with the ovum containing allele IO the offspring produced will have the genotype IO IO 8 and the phenotype is blood group O. 9 Three of the children who have the blood group O are produced this way and they have the genotype IOIO 10 When the sperm containing the IA allele fertilizes with the ovum containing allele IB then the offspring produced will have the genotype IA IB 11 and the phenotype is blood group AB. 12 One of the children who have the blood group AB is produced this way and has the genotype IA, IB. Any 10 points Genetic diagram: Marks

1 1 1 1 1 1 1

1 1 1

1 1 10

Parents Genotype PT 3,4

Ahmad IA I O

Amalina IB IO

PT 5., 6 Gametes Fertilisation

IA

IO

IB

IO

Offspring Genotype IAIB IOIO PT7.10 Phenotype blood group AB blood group B Pt 8, 11 (If answer using schematic diagram - only maximum 7 marks) No Criteria

Marks

11

PPDMT LONJAKAN SAUJANA SPM 2009

BIOLOGY P2

8b

Able to explain the effect of genetic engineering on man and the environment Suggested answer Good effects Agriculture F1 Genetic engineering used to produce disease resistant/ pest resistant plants e.g legumes, peas and beans P : Less pesticides are used - less pollution to the environment - better health for consumers. P : increase yield of crops - better livelihood for farmers. - help to solve problems of insufficient food. F2 : create crops with better nutrition value e.g tomatoes with higher vitamin A content - help to solve problems of malnutrition. F3: create crops with longer shelf lives e.g tomato - less food wastage F4 : genetically modified livestock e. g cows - produce meat with less fat / more milk. Medicine F5 : genetically modified bacteria produce insulin P ; for treatment of diabetis mellitus F6: Genetically modified yeast to produce vaccine for hepatitis P: for prevention of diseases. F7: Gene therapy for treatment of genetic disorders/ diseases e.g muscular dystrophy, rheumatoid arthritis, sickle cell anaemia P: Defective gene removed and normal gene inserted. Any 2F and P for agriculture, any 1F and P for medicine

Max : 6m

Bad effect F1 Pest resistant genes may be transferred to weeds P: may be difficult to control growth of weeds. F2: Some transgenic crops may have animal genes P : this may not be acceptable to certain groups for religious reasons. F3: Genetically modified foods may be harmful to health P: may activate human genes to cause cancer. F4: Transgenic organisms may affect the survival of other organisms in the ecosystem. P: may cause the imbalance of nature / ecosystem F5: Gene therapy used for the treatment of genetic disorder has its limitations. P : may not be acceptable because of religious and moral values. : very costly Any 2F and P Max 4m TOTAL QUESTION 9

4 10 20

12

PPDMT LONJAKAN SAUJANA SPM 2009

BIOLOGY P2

No

Criteria

Marks

(a)(i)

Able to discuss the importance of Biodiversity Suggested answer: F- it provides humans with necessities of life P- is a resource for food/timber to build shelter/as a fuel/fibres for clothing. F- many plant species are original sources of pharmaceutical drugs/medicines. P- new commodities, for example, new crop plants or medicinal materials could be developed using the gene pool from wild species in the forests. F- allow for biological control to maintain stable population P- regulate climatic conditions, biogeochemical cycles, prevent flooding F- Natural ecosystems and species in the wild are beautiful P- there is much pleasure to be derived from unspoilt natural environment F- ecotourism P- could provide income for some countries Any 4 points Max

(ii)

Able to discuss the Importance of mangrove swamps. Sample answer: F: resource for timber used in building industry P : has many varieties of species of mangrove trees F: Mangroves protect the coastlines and prevent coastal erosion. P: The roots of mangrove trees act as wave breakers which stabilise the coastlines. F:: Mangrove swamps are good breeding grounds for fish and crustaceans/ prawns, crabs etc. Important for fishermen P: The calm water and prop roots shelter the spawns from predators F: Serve as habitat for many species of birds, amphibians and reptiles. P: The habitat provides food, shelter, living space, nesting and breeding sites for these animals. F: Serve as natural barriers against torrential storms and tsunamis P: The trees block the water from flooding the land during a storm. Any 6 ponts

Max 6

13

PPDMT LONJAKAN SAUJANA SPM 2009

BIOLOGY P2

(b)(i)

Able to discuss the uses of microorganisms in ; 1 Waste treatment - rich in organic matters, bacteria and microorganisms - (in oxidation pond)the sewage is decomposed by(millions) of aerobic bacteria(in the presence of oxygen) - Decomposed sewage/sludge settled to the bottom of the pond fermentation takes place at sedimentation tanks using anaerobic bacteria produce methane/carbon dioxide/minerals digested sludge use as fertilizers Any 5 points 1 1 1 Max 5 1 1

(ii)

Food processing - F: Use of yeast in making of bread and cake - P: Fermentation by yeast produces carbon dioxide which makes dough rise. - F: Beer brewed from barley/ wine from grape juice - P: Yeast fermentation of the sugar in barley/ grape produces ethanol - F: Yoghurt is made from fermentation of milk by bacteria / Lactobacillus sp. / Streptooccus thermophillus - P: Bacteria converts sugar into lactic acid which coagulates the milk to / form yoghurt - F: Cheese made by adding bacteria and rennin to milk. - P: Milk separates into curd and whey/ coagulates - F: Soya sauce made from fermentation of soya bean by fungi - P: Yeast fermentation breaks down soya bean and gives it flavour Any 5 points

Max 5

TOTAL

20

14

j*k

j*k

j*k

j*k

j*k

j*k

j*k

j*k

j*k

j*k

j*k

j*k

j*k

j*k

j*k

j*k

j*k

j*k

j*k

j*k

j*k

j*k

j*k

j*k

j*k

j*k

j*k

j*k

j*k

j*k

j*k

j*k

j*k

j*k

j*k

j*k

j*k

j*k

j*k

j*k

j*k

j*k

j*k

j*k

j*k

j*k

j*k

j*k

j*k

j*k

j*k

j*k

j*k

MARKING SCHEME PAPER 1 TRIAL KEDAH 2010

1. 2. 3. 4. 5. 6. 7. 8. 9. 10. 11. 12. 13. 14. 15. 16. 17. 18. 19. 20. 21. 22. 23. 24. 25.

C A C D C B A D D B C D D D B D C A D C B D A B C

26. 27. 28. 29. 30. 31. 32. 33. 34. 35. 36. 37. 38. 39. 40. 41. 42. 43. 44. 45. 46. 47 48. 49. 50.

A A B C C C B A C D A A A D A A C B B A B D A C B

j*k

2 BIOLOGY SECTION A PAPER 2 [4551/2] Marking Criteria / Sample Answers Gills Tracheal system P : Filaments Q: Spiracles (R is ring of chitin which) support the tracheal / prevent the tracheal from collapsing. Diagram 1.1(b): P1: The filament have numerous thin-walled lamellae to maximise the surface area for gaseous exchange. P2: The gill filaments have thin membrane and covered by a net work of capillaries to transport respiratory gases. P3: The surface of the gills is moist which allows the gases to be dissolved. Any 1P Diagram 1.2(b) P1: The large number of tracheoles provides a large surface for the diffusion of gases. P2: Tip of tracheoles have thin permeable walls and contain fluid in which respiratory gases can dissolve. P3:Terminal ends of the tracheol remains moist which allows the gases to be dissolved. Any 1P (e) (i) P1:( The gaseous exchange process occurs over the whole body surface in an Amoeba sp) through simple diffusion. P2:Higher concentration of oxygen in the water surrounding causes oxygen to diffuse into the Amoeba. P3:Higher concentration of carbon dioxide in the cell causes carbon dioxide to diffuse out of the Amoeba. Any 2P S: Contractile vacuole 1 1 1 2 1 1 1 1 2 1 1

No. 1 (a) (i) (ii) (b)

Marks 1 1

2 1

(c)

(d)

(ii)

(iii) P1: Freshwater is hypotonic to the cytoplasmic fluid of Amoeba sp . P2: Water diffuses into the cell and fill the contractile vacuole by osmosis P3: When the contractile vacuole is filled with water to its maximum size, it contracts to expel its content from time to time. Any 2P

j*k

3 No. 2 (a) (i) (ii) Marking criteria/ Sample answers Osmosis P1 : Sucrose solution is hypertonic / more concentrated. P2 : Water diffuse from distilled water into the sucrose solution P3 : The level of sucrose solution in the capillary tube stop rising at the equilibrium stage / the concentration inside and outside of the visking tubing is the same / the amount of water diffuse into and out from the visking tubing is the same. Any 2 Ps F- Sucrose molecules are too large E- The visking tubing is a semi permeable membrane/ which only allows certain substances to pass through. (c) (i) Y : crenation Z : haemolysis 1 1 1 Mark 1

(b)

1 1 2

1 1 1 1 1 1 Any 3P

(ii) P1- Solution Z is hypotonic compare to red blood cell. P2- Osmosis occur P3- water leaves/ diffuses into the cell P4- Red blood cell expand/ swell and burst.

3 1 1 1

(iii) F : No P1 : Plant cell consists of cell wall P2 : Cell wall is made up of cellulose // Cell wall able to withstand the pressure. Any 2 Total

12

j*k

4 No. 3 (a) (i) (ii) Marking criteria/ Sample answers Absorption / Simple diffusion / facilitated diffusion
F1 thin wall/ one cell thick E1 increase rate of diffusion of digested food/ nutrients F2 large surface area/ has microvilli E2 increase rate of absorption of digested food/ nutrient

Mark 1 1 1 1 1 1 1 2 Any F + E

F3 has a network of capillaries/ blood vessels E3 to transport the absorbed nutrients

(b)

P: hepatic portal vein Q: lymphatic/lymph vessel/ duct P1: Deamination.// The amino group is removed (from amino acid)/ converted to ammonia . P2: (Ammonia) is converted to urea. P3: urea will be excreted through the kidneys. Any 2 Ps L1: A major energy reserve in the body// L2: (phospholipids are) components of the plasma membrane// L3: Lipids is used as a respiratory substrate// L4: Excess fats are stored in adipose tissues (under the skin, around internal organs) Any 1L A1:Amino acids are used in protein synthesis// A2:For repair and production of new protoplasm/growth and repair// A3:Used in the formation of enzymes/ some hormones/protein part of haemoglobin/ antibodies Any 1A G1:Glucose is used as the main respiratory substrate// It is oxidised to release energy (water and carbon dioxide)// G2:Excessive glucose is converted to glycogen // Blood glucose level rise / increase. Any 1 G

1 1 1 1 1 1 1 1 1

(c)

(d)

1 1 1

1 1 3 1 1

(e)

P1: Diabetes mellitus // Blood sugar level increases// Hyperglycemia P2: Excess glucose cannot be converted to glycogen. Total

2 12

j*k

5 No. 4 (a) Marking criteria/ Sample answers Mark

Both arrows correct (b) A Pulmonary artery B Pulmonary vein F : Contraction of ventricle / heart E1: generates a (high) pressure E2 : (to) propel/ force / pump the blood flow from the heart/ ventricle to vessel A Any two (d)(i) Coronary artery 1 1 1 1 1 1

(c)

(ii)

(e) (i)

P1: Cut the supply of O2/ nutrients to the heart muscle P2: causing chest pain / angina / heart attack / myocardial infarction Reject Heart problem P1: platelets break down and release chemicals P2: to cause platelets to stick to each other P3: platelets clump together to form a plug to prevent blood loss . P4: released thrombokinase and other clotting factors Any 2P

1 1 1 1 1 1

(ii)

P1 : Fibrinogen is soluble, fibrin is insoluble / not soluble P2 : Fibrin able to form fibres / meshwork / thread to trap blood cells, fibrinogen is not able to do so.

1 1

Total

12

j*k

No. 5 (a) (i) (a) (ii)

Marking Criteria / Sample Answers (Transfer/flow of) energy F : Phytoplankton is an autotrophic organism. P1 : Able to absorb light energy / consists of chloroplast. P2 : synthesis their own food / carry out photosynthesis Any 2 F1 : population of small fish increases P1 : no shark feed on small fish // shark is the predator F2 : population of plankton decreases P2 : more small fish feed on the plankton F3 : Eventually the population of small fish decreases Any 3 1 1 1

Marks 1

(b)

1 1 1 1 1

(c)

F : Commensalism P1 : Shark is the host / neither gain any benefit nor harmed. P2 : Remora benefits P3 : Remora obtain protection / food / transport from the shark. Any 3 P1 : Fertilizer washed away by rain water into the lake P2 : Nutrient / minerals content in the lake increase. P3 : alga bloom / alga grow rapidly in the lake. P4 : eutrophication occur. P5 : Oxygen content in the lake decrease / drop P6 : Fishes die / population decrease Any 3 P

1 1 1 1

(d)

1 1 1 1 1 1

Total

12

j*k

7 BIOLOGY SECTION A PAPER 2 [4551/2] - ESSAY


No. 6(a) (i) (a)(ii) Marking Scheme Continuous variation : body weight, height Discontinuous variation : types of earlobe, types of finger print. Continuous Variation Discontinuous variation P1 The changes of The differences among characteristics among individuals are distinct. individual are gradual P2 Continuous variation is Discontinuous variation is quantitative // characteristics qualitative // characteristic can be measured. is either present or absent. P3 The graph shows the normal The graph shows the distribution curve. discrete distribution. P4 The character is determined The character is determined by many genes by a single genes P5 The characteristic is The characteristic is influenced by the influenced by the genetic environmental factor and factor. genetic factor. P6 Exhibits a range of There are no intermediate phenotype with intermediate groups. characters. Albinisme F : Albinisme is caused by the change in gene // mutation P1 : Body / skin unable to produce black pigment (melanin) P2 : The skin and hair of albinos are white // their eyes are pink. Any 2 Sickle cell anaemia F : Sickle cell anaemia is caused by the change in the genes // mutation. P1 : haemoglobin produced is not normal / abnormal P2 : Abnormal haemoglobin unable to bind / transport / carries with oxygen efficiently. P3 : The patient will always feel weak / cannot carries out vigorous activities. Any 2 1 1 Mark

Any 4 pair

Max 4m

(b)

1 1 1

Max 2m

1 1 1 1 Max 2m

j*k

8
6(c) (i) Abiotic factors that cause variation between the two sets of ginger plants are: F1: Sun light P1: Plants need light energy to carry out photosynthesis for growth P2: Set A, plants are obtain more / exposed to sunlight // Plants in set B obtain less sunlight / not exposed to Sunlight. P3 : Growth rate of plants in Set A is higher than plants in Set B. F2: Space P4: Plants need (space) to grow a large root system / leaves P5: Plants able to absorb sufficient water and minerals/sunlight. P6: Set A, plants have larger space for the root and leaves to Grow // Plants in set B have smaller space for the root and leaves to grow. F3: Soil / minerals P7: Plants need mineral for (healthy) growth. P8: Loam soil provides more minerals in Set A. // Sandy loam soil in Set B contains less minerals. P9: Loam soil able to trap / store water better than sandy loam soil. Any 8 F1 : Plantlets from tissue culture have the same genetic material. P1 : This is to show /ensure/proof the differences of plants in Set A and Set B are not caused by genetic factor / have the same genetic material. // This is to show /ensure/proof the differences of plants in Set A and Set B are caused by abiotic factors.

1 1 1 1 1 1 1 1

1 1 1 1 1 1 max 8

6(c) (ii)

Total

20

No. 7(a)

Marking Scheme P1 : Nerve impulses arrive at the axon terminal of (presynaptic) neurone. P2 : Causes the synaptic vesicles to move towards the (presynaptic) membrane and fuse with the membrane. P3 : Neurotransmiters /acetylcoline (examples) molecules are released from synaptic vesicles. P4 : (The neurotransmitter molecules) diffuse across the synaptic cleft into the postsynaptic knob / dendrite / cell body of neighbouring neurone.. P5 : The neurotransmitter molecules bind to specific receptor sites in the postsynaptic knob. P6 : The binding triggers / generates new nerve Impulses. P7 : The impulses then move along the postsynaptic neurone. P8 : The release of neurotransmitter is in one direction, from the synaptic knob to the postsynaptic neurone. P9 : Mitochondria in the synaptic knob generate ATP / energy to synthesis neurotransmitter molecules. Any 6 1 1 1 1

Mark

1 1 1 1 1 Max 6

j*k

No. 7 (b)

Marking Scheme P1 : The receptor at the terminal of X stimulated by the heat. P2 : The receptor generates a nerve impulse. P3 : The nerve impulse travels along X / afferent neurone To the spinal cord. P4 : In the spinal cord, the nerve impulse is transmitted to an interneurone. P5 : From the interneurone, the nerve impulse is transmitted to an efferent neurone/ neurone Y. P6 : Nerve impulse travels along efferent neurone / Y and reach the effector / muscle tissue / fingers. P7 : Muscles contract to withdraw the hand / finger. Any 4 P1 : The receptors in the eyes detect the dog. P2 : Nerve impulses are generated and transmitted to the brain via the afferent neurone. P3 : The hypothalamus in the brain is stimulated. P4 : It actives the sympathetic nervous system to generate nerve impulses. P5 : Nerve impulses are transmitted to the adrenal medulla to stimulate secretion of adrenaline. P6 : Adrenaline carried / transported by blood circulatory system to the targeted organs. P7 : Adrenaline promotes the breakdown of glycogen to glucose. P8 : (Adrenaline) increases the breathing rate. P9 : More oxygen will be taken into the body P10 : (Adrenaline) increases the rate of heartbeat/ blood pressure. P11 : Rate of the blood flow increase. P12 : More glucose and oxygen will be supplied to the muscles. P13 : More energy produced by the muscles. // metabolic rate increase. P14 : Body has enough energy to face the fight or flight situation. Any 10 1 1 1 1 1 1 1 1 1 1 1 1 1 1 1 1 1 1 1 1 1

Mark

Max 4

7 (c)

Max 10

Total

20

j*k

10
No. 8 (a)(i) Marking Scheme P1 fish have streamline shapes // the anterior of the fish is smooth and rounded // the body is long and tapers towards the end. P2 the body of a fish is covered with scales that have a slimy coating P1 myotomes muscles are arranged in both side of the body P2 the vertebral column of the fish is flexible and can bent from side to side P3 myotome muscles act antagonistically in fish./ carry out opposite action in a fish P4 when the muscles on right side contract, the muscle on the left side relax P5 the tail/body will be bent to the right. P6 when the muscles on left side contract, the muscle on the right side relax P7 the tail/body will be bent to the left. P8 alternate contraction of the right and left myotome block enable its tail to move left and right P9 to produce a force that propel the fish forward. [ any 6] Similarities: F1 Both Joint S and Joint T has a cavity filled with svnovial fluid // lined with synovial membrane El Synovial fluid acts as lubricant to reduce friction between bones // absorbs shock of the movement. F2 The end surfaces of the humerus bone of Joint S and Joint T are covered with cartilage E2 To protect the bone / reduce friction between the bones F3 Both Joint S and T are connected with ligaments E3 to absorb shock // strengthen the articulation of bones/ joint. Differences: D1 Joint S is hinge joint E4 Joint S allows the movement of bones in one plane / direction D2 Joint T is ball-and-socket joint. E5 Joint T allows rotational movement of bones in all directions. [ any 8 ] Osteoporosis P1 : the bone become thinner / more brittle / porous / fragile. P2 : Loss of bone mass. P3 : Lack of calcium / phosphorus / vitamin D Arthritis P4 : Cartilage between bones become thinner. P5 : Ligaments become shorter / loss elasticity P6 : Less production of synovial fluid. P7 : The joints become swollen / stiff / painful [ any 4 ] Total 20 Mark 1

1 1 1 1 1 1 1 1 1 1

(a)(ii)

Max 6

(b)(i)

1 1 1 1 1 1

1 1 1 1 Max 8

8 (b)(ii)

1 1 1

1 1 1 1

Max 4

j*k

11
No. 9 (a) Marking Scheme The tree F1 : Less tree will be chopped / felled P1 : More CO2 absorbed by the trees for photosynthesis P2 : Avoid the increasing of CO2 in the atmosphere. P3 : Reduce the impact of Green house effect // global warming P4 : Less habitat of fauna and flora will be destroyed. P5 : Reduce / avoid the extinction of fauna and flora. P6 : To maintain / preserve the biodiversity. The oil / fuel // Save Energy F2 : Reduce the burning of oil / fuel P7 : More fuel/energy can be preserved for future. P8 : Less green house gases / acidic gases released. P9 : Reduce / avoid the impact of green house effect / acid rain. The Landfill F3 : Less landfill will be opened P10 : Landfill cause leaching / ground water pollution. P11 : Less diseases / health problem caused by the improper managed landfill. The Water F4 : Less used water / effluent / untreated sewage released into river. P12 : Reduce / avoid the impact of water pollution / avoid the extinction of aquatic organisms. Any 10 Good Effect G1 : Generate hydropower electricity G2 : As reservoir / to store water / supply fresh water G3 : Supply water for agricultural / industries. G4 : Place/site for recreation / tourism G5 : Reduce the flood problem at the downstream. Bad Effect B1 : Flooded / submerge trees / habitat of the fauna and flora B2 : Less tree / plants to carry out photosynthesis // Less CO2 absorbed for photosynthesis B3 : Amount of CO2 in the atmosphere increase B4 : Increase the impact of green house effect / global warming. B5 : Many species of fauna and flora extinct // Reduce the biodiversity. B6 : Reduce the flow of water at the downstream. B7 : Cause the population of aquatic life at the downstream reduce. B8 : Reduce the land used for residential / agricultural B9 : Flooded / destroy / loss of historical building / site. Any 10 Total 20 Mark

1 1 1 1 1 1 1 1 1 1 1 1 1 1

1 1

max 10

(b)

1 1 1 1 1 1 1 1 1 1 1 1 1 1

max 10

SULIT

4551/1

1. Which organ consists of cells which has the highest density of rough endoplasmic reticulum ? Organ manakah mengandungi sel-sel yang mempunyai kepadatan jalinan endoplasmik kasar paling tinggi? A B Stomach Perut Heart Jantung C Brain Otak D Kidney Ginjal

2.

Diagram 1 shows three types of cells. Rajah 1 menunjukkan tiga jenis sel.

Q Diagram 1

To which systems do the cells shown above belong? Kepada system manakah sel-sel di atas dipadankan ? P Digestive system Sistem pencernaan] Nervous system Sistem saraf Respiratory system Sistem respirasi Reproductive system Sistem pembiakan Q Respiratory system Sistem respirasi Reproductive system Sistem pembiakan Circulatory system Sistem saraf Digestive system Sistem pencernaan R Nervous system Sistem saraf Circulatory system Sistem peredaran Digestive system Sistem pencernaan Nervous system Sistem saraf

A. B. C . D .

[Turn over CONFIDENTIAL

SULIT

4551/1

3. Diagram 2 shows a unicellular organism living in freshwater pond. Rajah 2 menunjukkan sejenis organisma unisel yang hidup di dalam kolam air tawar.

Water Air
Diagram 2

Which process involves in the movement of water ? Apakah proses yang terlibat dalam pergerakan air. A B Diffusion Resapan Osmosis Osmosis C D Active transport Pengangkutan aktif Facilitated diffusion Resapan berbantu

4. Diagram 3 shows a cross section of a leaf. Rajah 3 menunjukkan keratan rentas daun
A B

Diagram 3 Which of the cell labelled A, B, C and D does not contain chloroplast? Antara sel yang berlabel A, B, C dan D yang manakah tidak mengandungi kloroplas?
[Turn over CONFIDENTIAL

SULIT 5. Diagram 4 shows a type of plant tissue. Rajah 4 menunjukkan sejenis tisu tumbuhan.

4551/1

Diagram 4 What is the importance of the thickening of substance X to the plant tissue? Apakah kepentingan penebalan bahan X terhadap tisu tersebut ? A B C D To transfer photosynthesis products Untuk memindahkan hasil fotosintesis To give turgidity to the tissues Untuk memberikan kesegahan kepada tisu To transfer water and mineral salts Untuk memindahkan air dan garam mineral To give support and mechanical strength Untuk memberikan sokongan dan kekuatan mekanikal

6. Which of the following sequence of organelles involved in the synthesis of extracellular enzymes is correct? Manakah di antara urutan berikut betul yang melibatkan organel dalam sintesis enzim luar sel? A Golgi apparatusRibosomes Rough endoplasmic reticulum Alat Golgi - Ribosom Jalinan endoplasmik kasar B Rough endoplasmic reticulumRibosomesGolgi apparatus Jalinan endoplasmik kasar Ribosom Alat Golgi C RibosomesGolgi apparatusRough endoplasmic reticulum Ribosom Alat Golgi Jalinan endoplasmik kasar D RibosomesRough endoplasmic reticulumGolgi apparatus Ribosom Jalinan endoplasmik kasar Alat Golgi]

[Turn over CONFIDENTIAL

SULIT

4551/1

7. Carrot slices are immersed in 0.1% sucrose solution. After 4 hours, the slices are found to be turgid and hard. Hirisan lobak merah direndam di dalam larutan sukrosa 0.1%. Selepas 4 jam, hirisan itu didapati segah dan keras. Which of the following statement explains this phenomenon? Antara pernyataan berikut, yang manakah menerangkan fenomena ini ? A The carrot cell wall prevent it from shrinking . Dinding sel karot menghalangnya dari mengecut. B The high concentration of the cell sap in the vacuole causes water to diffuse. . Kepekatan yang tinggi dalam sap sel vakuol menyebabkan air meresap ke dalam Sel. C The cell sap is hypotonic towards the sucrose solution. Sap sel adalah hipotonik kepada larutan sukrosa. D The carrot cell wall allows the sucrose molecules to diffuse into the cell. Dinding sel karot membenarkan molekul selulosa meresap ke dalam sel. 8. Diagram 5 shows a cell after immersed into a particular solution. Rajah 5 menunjukkan sel yang telah direndamkan ke dalam larutan tertentu.

Diagram 5 Which is experienced by the cell? Apakah yang dialami oleh sel itu ? A Crenation Krenasi B Plasmolysis Plasmolisis C Deplasmolysis Deplasmolisis D Haemolysis Hemolisis

[Turn over CONFIDENTIAL

SULIT 9. Diagram 6 shows the action of an enzyme on a substrate. Rajah 6 menunjukkan tindakan enzim ke atas suatu substrat.

4551/1

Diagram 6

What is represented by K? Apakah yang diwakili oleh K? A B Enzyme Enzim Enzyme-substrate complex Kompleks-enzim substrat C Products of reaction Hasil tindak balas D Substrate Substrat

10. Based on the information below, name the enzyme that can be used. Berdasarkan maklumat di bawah, namakan enzim yang sesuai digunakan. Extracting agar jelly from seaweeds Mengasingkan agar-agar daripada laut.] Removing the seed coats from cereal grains Mengeluarkan kulit dari bijirin A Zymase Zimase B Amylase Amilase C Celulase Selulosa D Protease Protease

11. Diagram 7 shows a graph between the rate of reaction at different substrate concentration when factor Q is changed. Rajah 7 menunjukkan graf diantara kadar tindak balas dan kepekatan substrat apabila faktor Q diubah.

[Turn over CONFIDENTIAL

SULIT

4551/1

Factor Q Faktor Q

Diagram 7 What is factor Q ? Apakah faktor Q ? A pH pH B Time Masa C Inhibitor Perencat D Enzyme concentration Kepekatan enzim

12. The following information shows starch molecules undergoing process M. Maklumat berikut menunjukkan molekul kanji melalui proses M. Process M Process M Starch ------------------ Maltose -------------------- Glucose [Kanj] Maltosa Glukosa What is process M ? Apakah proses M ? A Photosynthesis Fotosintesis B Hydrolisis Hidrolisis C Condensation Kondensas] D Polymerisation Pempolimeran

[Turn over CONFIDENTIAL

SULIT 13. Diagram 8 shows a cell cycle of an organism. Rajah 8 menunjukkan kitar sel bagi suatu organisma.

4551/1

Diagram 8 Which of the following represent X and Y ? Yang manakah di antara berikut mewakili X dan Y ? A B C D Phase X Meiosis Interphase Mitosis Interphase Phase Y Interphase Mitosis Interphase Meiosis

14. The diploid chromosomes in a leaf cell of a maize plant is 20. If one of the homologous chromosome pair does not separate during the Meiosis 1, how many chromosomes can be found in the male nucleus of a pollen grain of maize ? Nombor kromosom diploid dalam daun jagung ialah 20. Jika satu daripada Pasangan kromosom homolog tidak terpisah semasa Meiosis 1, berapakah bilangan kromosom yang mungkin didapati pada debunga daun jagung ? A 9 B 10 C 20 D 18

15. Which of the following statements explain the importance of mitosis to cells? Yang manakah di antara pernyataan berikut menerangkan kepentingan mitosis kepada sel ? I . To ensure the chromosomal number is constant in all somatic cells. Untuk memastikan bilangan kromosom adalah tetap dalam semua sel somatik. II To ensure the daughter cells have the same number of chromosomes as the parent cell. Untuk memastikan billangan kromosom adalah sama dengan biangan kromosom sel induk. III To ensure that the genetic material in the daughter cells is the same as in the parent cell Untuk memastikan bahan genetik sel anak dalah sama dengan sel induk.
[Turn over CONFIDENTIAL

SULIT IV To contribute to the genetic variation in the daughter cells. Untuk menyumbangkan kepada variasi genetik dalam sel anak. A I and II only B II and III only C D I , II and III only I , III and IV only

4551/1

16. Diagram 9 shows the correct proportion for the various classes of food in the food pyramid. Rajah 9 menunjukkan nisbah yang betul bagi pelbagai kelas makanan dalam piramid makanan.

Diagram 9

Which of the following shows the correct classes of food in the pyramid? Yang manakah di antara berikut menunjukkan kelas makanan yang betul dalam piramid makanan di atas? 1 Fats Carbohydrates Proteins Carbohydrates 2 Proteins Vitamins and minerals Carbohydrate s Fats 3 Carbohydrates Proteins Fats Vitamins and minerals 4 Vitamins and minerals Fats Vitamins and minerals Proteins

A B C D

[Turn over CONFIDENTIAL

SULIT 17. Diagram 10 shows the structure of the human alimentary canal. Rajah 10 menunjukkan struktur salur alimentari manusia.

4551/1

Diagram 10 Which organ produces the enzyme responsible for the breakdown of lipids ? Organ manakah merembeskan enzim yang bertanggungjawab untuk pemecahan lipid? A. S only B. P and Q C R and S D Q only

18. Which adaptations help the villi to absorb nutrients efficiently? Yang manakah penyesuaian vilus untuk menyerap nutrien secara berkesan? Abundant in number Bilangan yang banyak II Thin walls Dinding nipis III Having blood capillaries Mempunyai kapilari darah IV Lacteals to absorb fatty acids and glycerol Lakteal untuk menyerap asid lemak dan gliserol A B C D I and III only II and IV only I, II and III only I, II, III and IV I

19. When 0.4 g of groundnut is completely burnt, the temperature of 20 ml of water rise up from 30C to 70C. Calculate the energy value of the groundnut. (Specific heat capacity of water is 4.2 Jg C ) Bila 0.4g kacang tanah terbakar dengan lengkap, suhu 20 ml air meningkat daripada 30C kepada 70C. Hitung nilai tenaga kacang tanah ? ( Muatan haba tentu air ialah 4.2 Jg C ). A 1.4 kJg B 3.4kJg-1 C 8.4 kJg D 76.2 kJg
[Turn over CONFIDENTIAL

SULIT 20. Glucose + oxygen Glukosa + oksigen Carbon dioxide + water + 38 ATP Karbon dioksida + air + 38 ATP

4551/1

The above equation shows Persamaan di atas menunjukkan A. aerobic respiration respirasi aerobik B. anaerobic respiration respirasi anaerobik C condensation reaction tindakbalas kondensasi D hydrolytic reaction tindakbalas hidrolitik

21. Which of the following organelle involves in the gaseous exchange in Ameoba sp? Antara organel berikut yang manakah terlibat dalam pertukaran gas dalam Ameoba sp ? A Cell wall Dinding sel B Nucleus Nukleus C Cell membrane Membran sel D Vacuole Vakuol

22. Diagram 11 shows parts of the tracheal system of insect. Rajah 11 menunjukkan sebahagian daripada sistem trakea pada serangga.

tracheoles

body tissue

Diagram 11 What process occurs at X during the gas exchange of the insect? Apakah proses yang berlaku di X semasa pertukaran gas bagi serangga tersebut? A B Diffusion Resapan Osmosis Osmosis C Facilitated diffusion Resapan berbantu D Active transport Pengangkutan aktif
[Turn over CONFIDENTIAL

SULIT 23. .

4551/1

Diagram 12 shows three different types of interaction between organisms. Rajah 12 menunjukkan tiga jenis interaksi di antara organisma.

Diagram 12 Which of the following is true about the interactions K, L and M? Manakah di antara berikut benar tentang interaksi K , L dan M ? A B C D K Mutualism Mutualism Saprophytism Parasitism L Commensalism Parasitism Commensalism Commensalism M Parasitism Saprophytism Mutualism Mutualism

24. Which of the following chemical substance is used to kill or prevent the multiplication of microorganisms in the wound? Yang manakah di antara sebatian kimia berikut, digunakan untuk membunuh atau mencegah pembiakan mikroorganisma dalam luka ? A Antiseptic Antiseptik B Antibiotic Antibiotik C Vaccine Vaksin D Disinfectant Disinfektan

25. Nitrates and phosphates from farmland that flow into a lake caused rapid growth of algae . What is described by the above situation ? Nitrat dan fosfat yang dialirkan dari ladang ke dalam tasik telah menyebabkan pertumbuhan alga yang mendadak. Apakah yang diterangkan oleh situasi di atas ?. A Eutrophication Eutrofikasi B Pesticide pollution Pencemaran pestisid C Fertilizer accumulation Pengumpulan baja D Colonisation Pengkolonian
[Turn over CONFIDENTIAL

SULIT

4551/1

26. Diagram 13 shows plant cells. Rajah 13 menunjukkan sel tumbuhan. Which of the following cell is the product of meiosis? Sel yang manakah di antara berikut adalah produk pembahagian sel meiosis? A B

Diagram 13

27

Diagram 14 shows a part of a mangrove plant. Rajah 14 menunjukkan satu bahagian tumbuhan paya bakau.

What is structure S? Apakah struktur S? A. Succulent leaves Daun sukulen B. Pneumatophores Pneumatofor C. Vivipary seeds Biji benih vivipari D. Prop roots Akar jangkang

Diagram 14

[Turn over CONFIDENTIAL

SULIT

4551/1

28. Diagram 15 shows a stage in the blood clotting mechanism. Rajah 15 menunjukkan satu peringkat di dalam mekanisme pembekuan darah.

Diagram 15 Which of the following statement explains this stage? Antara pernyataan berikut, yang manakah menerangkan peringkat ini? A B C D Thromboplastin converts prothrombin to thrombin Tromboplastin menukarkan protrombin kepada trombin Thrombin converts fibrinogen to meshwork of fibrin. Trombin menukarkan fibrinogen kepada jaringan fibrin Platelets stimulate the formation of meshwork of fibrin. Platlet meransang pembentukan jaringan fibrin. Platelets release the thromboplastin to form meshwork of fibrin. Platlet membebaskan tromboplastin untuk membentuk jaringan fibrin.

29. Diagram 16 shows a cross-section through the carpel of a flower before fertilization. Rajah 16 menunjukkan keratan rentas melalui karpel bunga sebelum persenyawaan.

Where are the position of male and female gametes before fertilization? Di manakah kedudukan gamet jantan dan betina sebelum persenyawaan ? A B C D Male gamete 1 1 2 3 Female gamete 5 4 4 5

Diagram 16

[Turn over CONFIDENTIAL

SULIT 30. Diagram 17 shows part of the placenta. Rajah 17 menunjukkan bahagian plasenta.

4551/1

Diagram 17

In which parts do the blood contain the most oxygen and nutrients? Di bahagian manakah darah mempunyai kandungan oksigen dan nutrien yang tinggi? A B 1 and 3 1 and 4 C D 2 and 3 2 and 4

31. Diagram 18 shows the structure of a nephron. Rajah 18 menunjukkan struktur nefron.

Diagram 18

Which of the following activities cause X to be more permeable to water ? Yang manakah di antara aktiviti berikut menyebabkan X lebih telap kepada air ?
[Turn over CONFIDENTIAL

SULIT P - Drinking a lot of water Minum air dengan banyak Q - Eating salty foods Makan makanan yang banyak R - Not exercising Tidak melakukan senaman S - Playing sports Bersukan A P and R B P and S C D Q and R Q and S

4551/1

32. Diagram 19 shows the changes in the thickness of the uterus lining of a woman during her menstrual cycle. At which time is the woman most likely to be fertile? [Rajah 19 menunjukkan perubahan ketebalan lapisan uterus seorang wanita semasa kitar haid. Pada masa yang manakah wanita itu mengalami waktu paling subur?]

Diagram 19 33. The haemoglobin content of a pregnant mother is low. Which food should be taken to increase the haemoglobin content in her blood ? Kandungan haemoglobin seorang ibu mengandung adalah rendah. Makanan manakah yang perlu diambil untuk meningkatkan kandungan hemoglobin dalam darahnya? A Spinach Bayam C D Tomato Tomato Banana Pisang

B Potatoes Kentang

[Turn over CONFIDENTIAL

SULIT

4551/1

34. Diagram 20 shows a part of hind limp which consists of femur, tibia and fibula . Rajah 20 menunjukkan bahagian anggota belakang yang terdiri dari femur, tibia dan fibula. Patela

Diagram 20 Which of this action cannot be done if the patella is dislocated? Manakah tindakan berikut tidak berlaku jika patela beralih tempat? A B 35. Sitting down Duduk Sleeping Tidur C D Walking Berjalan Straightening the leg Meluruskan kaki

Diagram 21 shows the structures involved in reflex action. Rajah 21 menunjukkan struktur yang terlibat dalam tindakan refleks.

Diagram 21
Hot pan Periuk panas

Which of the following shows the correct sequence for the above action? Antara berikut, manakah menunjukkan urutan yang betul bagi tindakan di atas? A B PQRS P S R Q C D QRSP QSPR
[Turn over CONFIDENTIAL

SULIT 36.

4551/1

A farmer sprays all the mangoes in his farm with hormone X to ensure that all the mangoes ripen at the same time. What is hormone X? Seorang petani menyembur semua buah manggadi ladang nya dengan hormon X bagi memastikan semua mangganya masak pada masa yang sama. Apakah hormon X? A B Auxin Ethylene C Cytokinin D Gibberilin

37.

Diagram 22 shows the structure of human brain. Rajah 22 menunjukkan struktur otak manusia.

X Diagram 22

What is X? Apakah X? A B Cerebrum Serebrum Cerebellum Serebelum C D Spinal cord Saraf tunjang Medula oblongata Medula oblongata

38.

The following statements is about hormone X. Berikut adalah pernyatan tentang hormon X. Produced by corpus lutem and placenta [Dihasilkan oleh korpus luteum dan placenta] Promotes growth of endometrium and prevents menstruation. [Merangsang pertumbuhan endometrium dan menghalang haid]

[Turn over CONFIDENTIAL

SULIT What is hormone X? [Apakah hormone X?] A B 39. Oestrogen Estrogen Progesterone Progesteron C D Luteinising hormone Hormon pelutinan

4551/1

Follicle stimulating hormone Hormon perangsang folikel

Which of the following is true when the osmotic pressure in the blood decreases? Manakah di antara berikut benar sekiranya tekanan osmosis darah berkurangan? Secretion of ADH Rembesan ADH A B C D Increase Bertambah Decrease Berkurang Decrease Berkurang Increase Bertambah Reabsorption of water in kidney tubules Penyerapan air oleh tubul ginjal Increase Bertambah Decrease Berkurang Increase Bertambah Decrease Berkurang

40. Diagram 23 shows the stages in the development of follicle in the ovary of human. Rajah 23 menunjukkan peringkat perkembangan folikel dalam ovari manusia.

Diagram 23

What is the effect to the uterine wall when L developes into M? Apakah kesan kepada dinding uterus apabila L berkembang menjadi M?

[Turn over CONFIDENTIAL

SULIT A B It is repaired Ia diperbaiki It breaks down Ia terurai

4551/1

C It thickens Ia menebal D Its thickness maintains Ketebalannya dikekalkan

41. Melissa who is a carrier for colour blindness married to Aron a normal colour vision. What is the probability that their son is colour blind? Melisa adalah pembawa bagi buta warna berkahwin dengan Aron yang mempunyai penglihatan warna normal. Apakah kemungkinan anak lelaki mereka adalah buta warna ? A B 0% 25% C 50% D 100%

42. A pair of fraternal twins are brought up by two different families and have the following characteristics. Sepasang kembar seiras telah dibesarkan oleh dua keluarga yang berbeza dan mempunyai ciri seperti berikut. Nini Nina : Fat and fair complexion Gemuk dan kulit cerah : Thin and slightly dark complexion Kurus dan kulit agak gelap

Which factor causes the differences in the characteristics? Faktor yang manakah menyebabkan perbezaan ciri pada kembar itu? A B Genetic Genetik Environment Persekitaran C D Gene mutation Mutasi gen Chromosome mutation Mutasi kromosom

43. Diagram 24 shows a red rose plant is crossed with a white rose plant. The F1 generations that are produced are two red rose plants and two white rose plants. The allele for red rose plant, R is dominant to white rose plant, r. Rajah 24 menunjukkan pokok ros merah dikacukkan dengan pokok ros putih. Generasi F1 yang terhasil adalah dua pokok ros merah dan dua pokok ros putih. Alel untuk pokok ros merah, R adalah dominan kepada alel ros putih, r.
[Turn over CONFIDENTIAL

SULIT

4551/1

Parents Phenotype
[Fenotip induk]

X
Red Rose
[Ros merah]

White Rose
[Ros putih]

F1 generation Phenotype
[Fenotip generasi F1]

Red Rose
[Ros merah]

Red Rose
[Ros merah]

White Rose
[Ros Putih]

White Rose
[Ros Putih]

Diagram 24 What is the genotype of the parents? Apakah genotip bagi induk? A B C D Red Rose RR Rr Rr RR White Rose Rr Rr rr rr

[Turn over CONFIDENTIAL

SULIT 44. Diagram 25 shows ultrafiltration that occurs in the kidney. Rajah 25 menunjukkan ultraturasan yang berlaku dalam ginjal. Efferent arteriole Arteriol eferen

4551/1

Afferent arteriole Arteriol aferen Bowman capsule Kapsul Bowman

X Diagram 25 What are the substances that can move across X ? Apakah bahan yang dapat merentasi X? A B Fibrinogen Fibrinogen Leucocyte Leukosit C D Erythrocyte Eritrosit Amino acid Amino asid

45. Diagram 26 shows a shirt with a blood stain before and after being washed with detergent containing enzyme. Rajah 26 menunjukkan baju dengan kesan darah sebelum dan selepas dibasuh dengan pencuci mengandungi enzim.

Blood stain

Before Diagram 26

After

[Turn over CONFIDENTIAL

SULIT

4551/1

Which is the most suitable enzyme and temperature to give the result shown? Yang manakah enzim dan suhu yang paling sesuai untuk menghasilkan keputusan seperti di atas? Enzyme A B C D Lipase Protease Lipase Protease Temperature 37oC 18oC 18oC 37oC

46.

Diagram 27 shows a pair of chromosomes in a cell of an organism. Rajah 27 menunjukkan sepasang kromosom dalam sel suatu organisma. X

Diagram 27 What is X ? Apakah X ? A B Allele Gene C D Nucleotide Chromosome

47. Diagram 28 shows the regulation of human body temperature. Rajah 28 menunjukkan pengawalan suhu badan manusia. Normal body temperature Suhu badan normal Body temperature increase Suhu badan meningkat Temperature Regulation Centre Pusat kawalan suhu Diagram 28
[Turn over CONFIDENTIAL

Body temperature decrease Suhu badan menurun

Correction mechanism Mekanisme pembetulan

SULIT

4551/1

Which of the following correction mechanism occur ? Antara berikut yang manakah mekanisme pembetulan yang berlaku? l. Vasodilation Pemvasodilatan ll. Erector muscle contract Otot erektor mengecut] A B l and ll l, ll and lll C D III Vasoconstriction Pemvasocerutan lV Decrease in metabolic rate Kadar metabolisme menurun l and lV l, ll and lV

48. Diagram 29 shows the graphs of two types of variation . Rajah 29 menunjukkan graf untuk dua jenis variasi. Plant Tumbuhan Numbe r of individuals Bilangan individu Human Manusia

What type of variation shown in each population ? Apakah jenis variasi ditunjukkan dalam setiap populasi ? Human Manusia Continuous Continuous Discontinuous Discontinuous

Diagram 29

A B C D 49.

Plant Tumbuhan Discontinuous Continuous Discontinuous Continuous

The following food chain is found in a fresh water pond. Rantai makanan berikut terdapat dalam kolam air tawar. Phytoplankton water fleas fish Fitoplankton kutu air ikan Which of the following shows the relative amount of biological mass in the food chain ? Manakah di antara berikut menunjukkan amaun jisim biologi relatif dalam rantai
[Turn over CONFIDENTIAL

SULIT makanan ? Phytoplankton Fitoplankton 5g 30g 60g 60g Water flea Kutu air 15g 60g 5g 30g Fish Ikan 60g 15g 30g 5g

4551/1

A B C D

yang telah diberi suntikan vaksin sebanyak dua kali.

Diagram 30

Which of the following will be the type of immunisation acquired by these two patients? Antara berikut yang manakah merupakan jenis keimunan yang diperoleh oleh kedua-dua pesakit? A B C D P Artificial acquired active immunity Keimunan aktif buatan Artificial acquired passive immunity Keimunan pasif buatan Natural acquired passive immunity Keimunan pasif semulajadi Natural acquired active immunity Keimunan aktif semulajadi Q Artificial acquired passive immunity Keimunan pasif buatan Artificial acquired active immunity Keimunan aktif buatan Natural acquired active immunity Keimunan aktif semulajadi Natural acquired passive immunity Keimunan aktif semulajadi

KERTAS SOALAN TAMAT.


[Turn over CONFIDENTIAL

MARKING SCHEME BIOLOGY 1 (4551/1) SPM TRIAL EXAMINATION 2009 1. 2. 3. 4. 5. 6. 7. 8. 9. 10 . 11 . 12 . 13 . 14 . 15 . 16 . 17 . 18 . 19 . 20 . 21 . 22 A B B A D D B D C C D B B A C B D D C A C A 26. 27. 28. 29 30. 31. 32. 33. 34. 35. 36. 37. 38. 39. 40. 41. 42. 43. 44. 45. 46. 47. B C B D B D B A C C B B B A C C B C D D A B

23 . 24 . 25 .

C A A

48. 49. 50

D D A

Section A Bahagian A [60 marks] [60 markah] Answer all questions in this section. Jawab semua soalan dalam bahagian ini. 1 Diagram 1 shows the structure of a plasma membrane. Rajah 1 menunjukkan struktur membran plasma.

For examiners use

Q Diagram 1 Rajah 1 (a)

Name the parts labelled Q and R. Namakan bahagian yang berlabel Q dan R. Q : ________________________________________________________ R : ________________________________________________________ [2 marks]
1(a)

(b)(i)

State the component of structure P. Nyatakan komponen struktur P. ___________________________________________________________ [1 mark ]

1(b)(i)

(ii) Explain the main function of P. 2 [Lihat sebelah

Terangkan fungsi utama P. ___________________________________________________________ ___________________________________________________________ [1 mark ] (c) The plasma membrane is said to be semi-permeable. What is the meaning of semi-permeable? Membran plasma dikatakan bersifat separa-telap. Apakah yang dimaksudkan dengan separa-telap? ______________________________________________________________ ______________________________________________________________ [ 1 mark]
1 (c) 1(b)(ii)

Diagram 1.2 Rajah 1.2 d) Graph in Diagram 1.2 shows the percentage of red blood cells that are burst or shrink when placed in salt solution of different concentration. Graf dalam Rajah 1.2 menunjukkan peratus sel darah merah yang pecah atau mengecut apabila dimasukkan ke dalam larutan garam yang berbeza kepekatan. (i) Based on the graph given, state the concentration which is isotonic to blood plasma. 3 [Lihat sebelah

Berdasarkan graf yang diberi, nyatakan kepekatan larutan yang isotonik terhadap plasma darah. _________________________________________________________ (1 mark) (ii) Explain your answer in (b)(ii). Terangkan jawapan anda dalam (b)(ii). ________________________________________________________ _ (1 mark) (iii) Comment on the osmotic pressure at Q. Berikan ulasan tentang tekanan osmosis di Q.
________________________________________________________________ ________________________________________________________________ ________________________________________________________________ ________________________________________________________________

1(d)(i)

1(d)(ii)

1(d)(iii)

[ 2 marks] (e) The concentration of ions inside root cells is up to 100 times greater than in the soil. Anyway, the ions are still transported into the cells by active transport. Kepekatan ion di dalam sel akar adalah 100 kali lebih tinggi berbanding di dalam tanah. Walau bagaimanapun, ion-ion tersebut masih diangkut ke dalam sel secara pengangkutan aktif. (i) Define active transport. Takrifkan pengangkutan aktif.
___________________________________________________________ ___________________________________________________________ 1 (e)(i)

[ 1 mark]

4 [Lihat sebelah

(ii)

Explain what will happen to the uptake of the ions by root cells if the roots are immersed in a solution containing metabolic poisons such as cyanide. Terangkan apa akan berlaku terhadap pengangkutan ion oleh oleh sel akar jika akar tersebut direndam di dalam larutan yang mengandungi racun metabolik seperti sianida.
___________________________________________________________ ___________________________________________________________ ___________________________________________________________ ___________________________________________________________ [3 marks] 1(e)(ii)

TOTAL

5 [Lihat sebelah

Diagram 2.1 shows the biochemical processes involve molecule K, enzyme L and molecule M occur in organ X and organ Y. Rajah 2.1 menunjukkan proses biokimia yang melibatkan molekul K, enzim L dan molekul M yang berlaku di dalam organ X dan organ Y.
M u s c le c e lls

For examiners use

O rg a n X h e p a tic p o r ta l v e in K M
+

O rg a n Y

+ ......

E nzym e L

G ly c o g e n

Diagram 2.1 Rajah 2.1 (a)(i) Name organ X and organ Y. Namakan organ X dan organ Y. Organ X : ___________________________________________________ Organ Y : ___________________________________________________ [ 2 marks ] (ii) Name molecule K, molecule M and enzyme L. Namakan molekul K, molekul M dan Enzim L. Molecule K / molekul K : Molecule M / molekul M : _______________________________________ Enzyme L / enzim L : (b) _______________________________________ [ 3 marks ] _______________________________________ 2(a)(ii) 2(a)(i)

State two characteristics of enzyme L based on Diagram 2.1. Nyatakan dua ciri enzim L berdasarkan Rajah 2.1. ____________________________________________________________ ____________________________________________________________ ____________________________________________________________ 6 [Lihat sebelah 2 (b)

[ 2 marks ] (c) Molecules M are transported from organ Y to muscle cells. Explain why molecule M is needed in muscle cells. Molekul M diangkut dari Organ Y ke sel-sel otot. Terangkan kenapa molekul M diperlukan di dalam sel-sel otot. ____________________________________________________________ ____________________________________________________________ ____________________________________________________________ ____________________________________________________________ [ 3 marks ] (d) Explain the importance of forming glycogen. Terangkan kepentingan pembentukan glikogen. ____________________________________________________________ ____________________________________________________________ ____________________________________________________________ [ 2 marks ]
TOTAL

2 (c)

2 (d)

7 [Lihat sebelah

R
Soil

S T
For examiners use

Diagram 3.1 shows three stages X, Y and Z in meiosis. Rajah 3.1 menunjukkan tiga peringkat X, Y dan Z dalam meiosis.

Y Diagram 3.1 Rajah 3.1

(a)(i)

Name stages X and Y. Namakan peringkat X dan Y. X : ______________________________________________________ Y : ______________________________________________________ [2 marks] 3(a)(i) (

(ii) State two differences between chromosomal behaviour at X and Y. Nyatakan dua perbezaan perlakuan kromosom di X dan Y. ____________________________________________________________ ____________________________________________________________ ____________________________________________________________ [ 2 marks ] (b) i) State the occurrence at Z. Nyatakan kejadian yang berlaku di Z. 3(b)(i) ____________________________________________________________ ___________________________________________________________ ____________________________________________________________ [ 2 marks ] 8 [Lihat sebelah 3(a)(ii)

ii)

The chromosome number in somatic cell of this organism is 12. State the chromosome number in each of the daughter cell in Z. Give a reason for your answer. Bilangan kromosom dalam sel soma bagi organisma ini ialah 12. Nyatakan bilangan kromosom dalam setiap sel anak Z. Berikan alasan anda. ______________________________________________________ ______ ____________________________________________________________

3(b)(ii)

____________________________________________________________ [ 2 marks ] (c) Diagram 3.2 shows the process of sperm formation in the human testis. Rajah 3.2 menunjukkan proses pembentukan sperma di dalam testis manusia.

Diagram 3.2 Rajah 3.2 (i) Are cells A, cell B and cell C genetically identical? Explain. Adakah sel A, sel B dan sel C seiras dari segi genetic? Terangkan. ____________________________________________________________ ____________________________________________________________ ____________________________________________________________ 9 [Lihat sebelah 3(c)(i)

(ii)

____________________________________________________________ [2 marks] If Cell B undergoes an improper cell division, cell D might receive an extra chromosome. State the number of chromosomes in Cell D. Jika Sel B melalui pembahagian sel yang tidak sempurna, sel D berkemungkinan menerima satu kromosom tambahan. Nyatakan bilangan kromosom di dalam sel D. __________________________________________________________ [ 1 mark ]

3(c)(ii)

(iii) If cell D fertilises with an ovum, the zygote formed might develop into an abnormal male. State the syndrome of the individual. Jika sel D bersenyawa denngan ovum, zigot yang terbentuk akan berkembang menjadi lelaki yang abnormal. Nyatakan sindrom individu tersebut. __________________________________________________________ [ 1 mark ]

3(c)(iii)

TOTAL

For examiners use

10 [Lihat sebelah

Diagram 4.1 shows the apparatus set up in an experiment to study the role of the vascular tissue in the transport of water in plants. Rajah 4.1 menunjukkan susunan radas eksperimen untuk mengkaji peranan tisu vaskular dalam pengangkutan air di dalam tumbuhan.

4(a) Diagram 4.1 Rajah 4.1 (a) State the function of the eosin solution. Nyatakan fungsi larutan eosin. ___________________________________________________________ [ 1 mark ] (b) The stem of the plant is cut across at XY and viewed under a microscope. A cross section of the stem is shown in Diagram 4.2. Akar tumbuhan tersebut dikerat secara merentas pada XY dan diperhatikan di bawah mikroskop. Keratan rentas akar ditunjukkan dalam Rajah 4.2. K

Diagram 4.2 Rajah 4.2 11 [Lihat sebelah

4(b)

Name the parts labelled K and M. Namakan bahagian yang berlabel K dan M. K M (c) : : ___________________________________________________ ___________________________________________________ [ 2 marks ] 4(c)

Name the tissue which is responsible for transporting water and minerals ions from the roots to the upper parts of the plant. Namakan tisu yang terlibat dalam pengangkutan air dan ion mineral dari akar ke bahagian atas tumbuhan. ____________________________________________________________ [ 1 mark ]

(d)

If the root of the plant is cut across, draw and label the observation made. Jika keratan rentas dibuat ke atas akar tumbuhan, lukis dan labelkan pemerhatian anda.

4(d)

[ 3 marks ]

For examiners use

(e)

Diagram 4.3 shows the effect of removing tissue M from the stem. 12 [Lihat sebelah

4(e)(i) Diagram 4.3 Rajah 4.3 (i) State the type of transport involved in Diagram 4.3. Nyatakan jenis pengangkutan yang terlibat dalam Rajah 4.3. ___________________________________________________________ [ 1 mark ] (ii) Explain why does the part above the ring become swollen after two weeks. Terangkan mengapa bahagian atas gelang membengkak selepas dua minggu. ___________________________________________________________ ____________________________________________________________ ___________________________________________________________ [2 marks] (iii) Explain why have the leaves not wilted after two weeks. Terangkan mengapa daun-daun tidak layu selepas dua minggu. ___________________________________________________________ ____________________________________________________________ ___________________________________________________________ [2 marks] 4(e)(ii)

4(e)(iii)

TOTAL

13 [Lihat sebelah

5.

Figure 5 shows the reflex arc that occurs when the finger is accidentally pricked with a needle. Rajah 5 menunjukkan suatu arka refleks yang berlaku apabila jari tangan secara tidak sengaja dicucuki oleh sebatang jarum.

5(a)

Diagram 5 Rajah 5 (a) Complete the above figure by drawing the appropriate neurones involved in the reflex action. Lengkapkan rajah di atas, dengan melukis neuron yang terlibat di dalam tindak balas refleks. [2 marks] (b) Explain the transmission of impulse from one neurone to another neurone. Terangkan pemindahan impuls dari satu neuron ke neuron yang berikutnya.
________________________________________________________________ ________________________________________________________________ ________________________________________________________________ ________________________________________________________________ ________________________________________________________________ ________________________________________________________________

5(b)

5(c)

[ 4marks ] 14 [Lihat sebelah

(c)

Name the structures M and N. Namakan struktur M dan N. 5(d) M : ____________________________________________________ N : ____________________________________________________ [2 marks] 5(e)

(d)

Differentiate between the above reflex action with the voluntary action. Bezakan di antara tindakan refleks di atas dengan tindakan terkawal. ___________________________________________________________ ___________________________________________________________ ___________________________________________________________ _ [1 mark]

(e)

State the importance of reflex action to us. Nyatakan kepentingan tindakan refleks kepada kita. ____________________________________________________________

5(f)

(f)

[1 mark ] If efferent neurone is injured and damaged, predict what will happen to the person. Jika neuron eferen cedera dan rosak, ramalkan apa yang akan berlaku Kepada orang tersebut. ____________________________________________________________ ____________________________________________________________ [1 mark ]

TOTAL

Section B Bahagian B [40 marks] 15 [Lihat sebelah

Answer any two questions from this section. Jawab mana-mana dua soalan daripada bahagian ini 6(a) Figure 6.1 shows movement activities in a human. Rajah 6.1 menunjukkan aktiviti pergerakan pada manusia.

(i) Diagram 6.1

(ii)

. Based on Figure 6.1(i) and Figure 6.1(ii), explain how the above movement takes place which involves muscles, tendons, bones, ligaments and joints Berdasarkan Rajah 6.1(i) dan Rajah 6.1(ii), terangkan bagaimana pergerakan di atas berlaku yang melibatkan otot, tendon , tulang, ligamen dan sendi. [10 marks] (b)(i) By giving one example of woody plant and and non-woody, explain how the support system in woody plants differs from that of non-woody plants. Dengan menyatakan contoh, terangkan bagaimana sistem sokongan pada tumbuhan berkayu berbeza daripada tumbuhan tidak berkayu. [10 marks]

7(a)

Diagram 7.1 shows how the respiratory gases are transported in the human body. Rajah 7.1 menunjukkan bagaimana gas respirasi diangkut dalam badan manusia. 16 [Lihat sebelah

Diagram 7.1 (i) Based on Diagram 7.1, explain how the transport of oxygen and carbon dioxide takes place in the body cells Berdasarkan Rajah 7.1, terangkan bagaimanakah pengangkutan oksigen dan karbon dioksida berlaku di dalam sel-sel badan. [6 marks] Describe the adaptations of the alveolus for gaseous exchange. Terangkan penyesuaian pada alveolus yang membolehkannya utuk melakukan proses pertukaran gas. [4 marks]

(ii)

(b)

The shaded area of the graph in Diagram 7.2 shows the intake of oxygen by an athlete before, during and after running for five minutes. Kawasan yang berlorek pada graf dalam Rajah 7.2 menunjukkan pengambilan oksigen oleh seorang atlet sebelum, semasa dan selepas 17 [Lihat sebelah

berlari selama 5 minit.

Diagram 7.2 Based on the graph, explain how an oxygen debt is built up when an athlete is running and how it is settled after he stops running. Berdasarkan graf, terangkan bagaimana hutang oksigen terhasil semasa atlet itu berlari dan bagaimana ia diselesaikan selepas beliau berlari. [10 marks]

8(a)

Mr Nick has group A blood while his wife has group B blood. The group of their son is O. Explain how this happen. 18 [Lihat sebelah

Mr Nick mempunyai kumpulan darah A manakala isterinya mempunyai kumpulan darah B. Kumpulan darah anak lelaki mereka ialah O. Terangkan bagaimana ini boleh berlaku. [10 marks] (b) Nowadays, the DNA fingerprinting technique has replaced the common fingerprinting technique in criminal investigations. Pada masa kini, teknik cap jari DNA telah menggantikan teknik cap jari biasa dalam penyiasatan jenayah. (i) Explain how DNA fingerprinting is carried out. Terangkan bagaimana cap jari DNA dilakukan [4 marks] (ii) Based on the given statement, state your opinion an the advantages and disadvantages of DNA fingerprinting. Berdasarkan pernyataan yang diberikan, nyatakan pendapat kamu tentang kebaikan dan keburukan cap jari DNA. [6 marks]

9(a)

Diagram 9 shows a mangrove swamp. Rajah 9 menunjukkan kawasan paya bakau.

19 [Lihat sebelah

Diagram 9 (i) Explain why most plants cannot colonise and grow in the swamps. Terangkan mengapa kebanyakan tumbuhan tidak boleh hidup dan tumbuh di kawasan paya bakau. [5 marks] Explain how the mangrove trees adapt themselves to the harsh living conditions. Terangkan bagaimana pokok bakau ini menyesuaikan diri dengan keadaan hidup yang sukar. [5 marks]

(ii)

(b)

Development that is not planned and managed properly has brought negative effects to the ecosystem such as land erosion, flash flood, landslides, global warming, thinning of the ozone layer, climate change and the extinction of certain species. Pembangunan yang tidak terancang dan terurus dengan teliti boleh membawa kesan negatif kepada ekosistem seperti hakisan tanah, banjir kilat, tanah runtuh, pamanasan global, penipisan lapisan ozon, perubahan iklim dan kepupusan spesis tertentu. Based on the above statement, describe the effects of unplanned development and improper management of the ecosystem. Berdasarkan kenyataan di atas, terangkan kesan pembangunan yang tidak terancang dan terurus dengan teliti kepada ekosisitem. [10 marks]

20 [Lihat sebelah

MARKING SCHEME BIOLOGY 2 (4551/2) SPM TRIAL EXAMINATION 2009

No. 1(a)

Marking Criteria Able to name the parts labelled Q and R. Sample answer : Q : Carrier protein R : Channel protein / pore protein 1 1

Mark

1(b)(i)

Able to state the component of structure P. Sample answer : It is composed of two layers of phospholipids 1

(ii)

Able to explain the main function of P. Sample answer : Acts as a barrier between the internal and external environment of the cell // Allows only specific molecules to pass through it // provide the structural basis for all cell membrane.

1(c)

Able to give the meaning of semi-permeable. Sample answer : A semi-permeable plasma membrane is a membrane that allows only certain substances to move freely across it. 1 1

1 (d)(i)

Able to state the concentration which is isotonic to blood plasma. Sample answer : 0.45 g/100 cm3 1

1(d)(ii)

Able to explain the answer in (d)(i).

SampleAnswer : Both percentage of haemolysis of red blood cells and percentage of crenation of red blood cells are zero (0%). 1(d)(iii) Able to comment on the osmotic pressure at Q. SampleAnswer : F : The osmotic pressure inside the red blood cells is equivalent to its environment. P2 : Amount of water moving in and out of the cells are the same, P3 : therefore the size and structure of the red blood cells does not change. ( F + Any P2/P3 ) 1(e)(i) Able to define active transport. Sample answer : Active transport is a movement of substances / molecules / ions against the concentration gradient / from low to high concentration across the plasma membrane with the help of carrier protein and energy / ATP. (ii) Able to explain what will happen to the uptake of the ions by root cells if the roots are immersed in a solution containing metabolic poisons such as cyanide. Sample answer : P1 there is no uptake of ions by root cells P2 metabolic poisons kill/ damaged the (root) cells P3 no energy/ ATP is produced P4 active transport does not occur (Any three) 1 1

TOTAL

13 marks

2(a)(i)

Able to name organ X and organ Y. Sample answer : Organ X : Ileum // small intestine Organ Y : Liver 1 1

(ii)

Able to name molecule K, molecule M and enzyme L. Sample answer : Molecule K : Starch Molecule M : Glucose Enzyme L : (Pancreatic) Amylase 1 1 1

(b)

Able to state two characteristics of enzyme L based on Diagram 2.1 Sample answer : 1. Enzyme remains unchanged at the end of the reaction (and can be used again). 2. Enzyme is substrate specific / reaction is very specific 1 1 2

(c)

Able to explain why molecule M is needed in muscle cells. Sample answer : Pt. 1 Molecule M / glucose is the substrate for respiration Pt. 2 As the muscle cells contract and relax, energy is needed for activities Pt. 3 therefore, molecule M is needed in muscle cells to provide energy from respiration process. 1 1 1 3

(d)

Able to explain the importance of forming glycogen. Sample answer : Pt.1 : Glycogen is the main reserve of carbohydrates in animals 1

Pt. 2 It can be converted back to glucose when energy is needed from respiration process

1 TOTAL

2 12 marks

3(a(i))

Able to name stage X and Y. Sample answer : X : Prophase I Y : Metaphase I 1 1 2

(ii)

Able to Able to state two differences between chromosomal behaviour at X and Y. Sample Answer:

1 1

1 2 (b)(i)

1 1

(ii)

Prophase I Metaphase I (Paired homologous chromosomes) are arranged randomly. (Paired homologous chromosomes) are arranged on the metaphase plate / equatorial plane. Spindle fibre does not hold on the centromere of the chromosomes . Spindle fibre holds on the centromere of the chromosomes.

1 1 2

(c)(i)

1 1 2

(ii)

(The homologous chromosomes paired and) crossing over take place. (The homologous chromosomes paired) crossing over does not take place.

1 1 2

(iii)

( Any 2 ) Able to state the occurrence at Z. Sample Answer:


P1 : Four daughter cells formed P2 : Each daughter cell has two chromosomes / haploid / n

1 TOTAL 12 marks

Able to state the chromosome number in each of the daughter cell in Z and able to give reason. Sample answer :

P1 : 6 (chromosomes). P2 :(During meiosis) the daughter cell receives half the number of chromosome from the parent cell / 2n // Daughter cell haploid / n, parent cell diploid / 2n Able to state either cell A, cell B and cell C are genetically identical and explain. Sample answer : F : Cell A is similar to cell B but is different from cell C. P : Cell A and cell B are products of mitosis whereas cell C is a product of meiosis. Able to state the number of chromosome in Cell if Cell B undergoes an improper cell division. Sample answer : 24 (chromosomes) Able to state the syndrome of the individual. Sample answer : Downs syndrome // Klinefelters syndrome 4(a) Able to state the function of the eosin solution. Sample answer : To stain the xylem (vessels) (with red dye) 4(b) Able to name the parts labelled K and M. Sample answer : K : Xylem M : Phloem 4(c) Able to name the tissue which is responsible for transporting water and mineral ions from the roots to the upper parts of the plant. Sample answer : 1 1 2 1 1

Xylem 4(d) Able to draw and label the observation of the root cut across. Sample answer : Xylem

Phloem

Pericycle Cortex // ground tissue Drawing 1 m Any 2 labels 2 m 1 2

(e)(i)

Able to state the type of transport involved in Diagram 4.3. Sample answer : Translocation 1

(ii)

Able to explain why does the part above the ring become swollen after two weeks. Sample answer : F : The products of photosynthesis cannot be transported to the parts below the ring P : as tissue M / phloem is removed

1 1

(iii) Able to explain why have the leaves not wilted after two weeks. Sample answer : F : Water can still be transported to the leaves P : as tissue K / xylem is not removed from the stem 1 1

TOTAL

12 marks

5(a)

Able to complete the drawing the appropriate neurons involved in the reflex action. Sample answer :

3 neurones 2 m 2 neurones - 1 m 5(b) Able to explain the transmission of impulse from one neurone to another neurone. Sample answer : Pt..1 When an impulses arrives in the axon terminal Pt. 2 it stimulates (synaptic) vesicles to move towards and bind with the presynaptic membrane Pt. 3 The vesicles fuse / release the neurotransmitter into the synapse Pt. 4 The neurotransmitter molecules across the synapse to the dendrite of another neurone Pt. 5 Stimulated to trigger a new impulses which travels along the neurone ( Max 4 ) 5(c) Able to name the structure M and N. Sample answer : M : Sensory reseptor // finger tip N : Effector // muscles tissues

1 1 1 1 1 4

1 1

5(d)

Able to differentiate the reflex action with the voluntary action. Sample answer : The reflex action is governed by the spinal chord whereas the voluntary action is governed by the cerebrum. 1 1

5(e)

Able to state the importance of reflex action to us. Sample answer : To protect the body against injuries 1 1

5 (f)

Able to predict the effect on O if it is injured or damaged. Sample answer: 1. The nerve impuls will be sent from afferent neurone to the effector 2. The effector / muscles will not contract 3. The hand will not be removed immediately from the needle. (Any one ) 1 1 1 1

TOTAL

11 marks

6(a)

Able to relate the tissues involved in producing the running movement Sample Answer: Pl- Tendons, ligaments, bones, muscles and joints are important features in a movement, P2- Tendons connect muscles to bones P3- Tendons are strong and non elastic P4- Force is transferred to bones through tendons. P5- Movement at the joint is possible with the aid of ligaments. P6- Ligaments connect two bones together P7-to give support and strength to the joint. P8- Ligaments are strong and elastic. P9- The quadriceps / extensor muscles contract while the biceps femoris muscles relax and the leg is straightened. P10- The biceps femoris muscles contract while the quadriceps / extensor muscles relax and the leg is bent. P11- Calf muscles contract to lift up the heels. P12-Feet push downward and backward P13-Repeated contraction and relaxation of muscles result in the running movement. MAXIMUM: 10 marks 1 1 1 1 1 1 1 1 1 1 1 1 10

(b) Able to give example and explain how the support system in woody plants differs from that of non-woody plants. Examples 2 marks , Facts 8 marks Sample answers: Non-woody plants (herbaceous plants) Example: Balsam plant/ any suitable answer P1: (Support in herbaceous plants is) provided by the turgidity of the parenchyma / collenchyma cells 1 1 1

P2: (When there is enough warm in the ground). the cells take in water by osmosis and become turgid. P3: The turgor pressure of the fluids in the vacuoles pushes the cell contents / plasma membrane against the cell wall P4: creating support for it stem/ roots /leaves P5: The thin thickening die cell walls with cellulose / collenchyma cells gives support to herbacous plants Woody plants : Example : Rambutan tree/ hibiscus/ any suitable example P6: Woody plants have specialised tissues/ sclerenchyma tissues/ xylem vessels / tracheids. to give them support; P7: These tissues have cellulose walls which have deposits of lignin for added strength. P8: Sclerenchyma cells have very thick walls (which do not allow water to pass through). P9: (These cells are dead cells and) their function is to provide support for the plant. P10: Xylem vessels have thick walls of lignin which are deposited during the plant's secondary growth. P11: The lignified xylem vessels form the woody tissues of the stem. P12: This makes the plant stronger and also provides support for the plant. P13: Tracheids are also dead cells with thick walls and very small diameters. P14: They are found with the xylem vessels and together they support the plants. MAXIMUM: 10 marks

1 1 1 1 1 1 1 1 1 1 1 1 1

10

20 marks

TOTAL 7(a) (i) Able to explain how the transport of oxygen and carbon dioxide takes place in the body cells Sample answers: P1: The blood circulatory system transport oxygen from the alveoli to the body cells. P2: Oxygen combines with the haemoglobin in the red blood cells P3: to form oxyhaemoglobin (which is unstable.) P4: Oxygen is carried (in form of oxyhaemoglobin) to the tissues (which have a low partial pressure of oxygen.) P5: The (unstable) oxyhaemoglobin breaks down into oxygen and haemoglobin again. P6: Oxygen (molecules are) transferred to the body cells P7: Carbon dioxide binds (itself) to the haemoglobin P8: (and is) transported in the form of carbaminohaemoglobin. P9: Carbon dioxide is (also) transported as dissolved carbon dioxide (in the blood plasma.) P10: Most of carbon dioxide is carried as bicarbonate ions (dissolved in the blood plasma.) P11: When the blood carrying carbon dioxide reaches the body cells, the carbon dioxide diffuses into the blood plasma and combines with the red blood cells. P12:Carbon dioxide reacts with water to form carbonic acid. P13:Carbonic anhydrase in the red blood cells catalyse the formation of carbonic acid. P14: The carbonic acid then dissociates into a hydrogen ions and bicarbonate ions. MAXIMUM: 6 marks (ii) Able to describe the adaptations of the alveolus for gaseous exchange Sample answer: F1: The millions of alveoli P1: provide a large surface area for gaseous exchange. F2: The walls of the alveoli are moist P2: and this allows respiratory gases to dissolve easily to them. F3: The walls of the alveoli are very thin (one-cell thick) P3: forquick / easy diffusion of gases. F4: The alveoli are richly supplied with blood capillaries 1 1 1 1 1 1 1 1 1 1 1 1 1 1 6

P4: to increase the rate of diffusion / the rate of the transportation of gases MAXIMUM: 4 marks (b) Able to explain how an oxygen debt is built up when an athlete is running and how it is settled after he stops running. Sample answer: P1: During a vigorous exercise /running, the breathing rate is increased. P2: This is to supply more oxygen (quickly to the muscles) P3:for rapid muscular contraction). P4: However, the supply of oxygen to muscles is still insufficient P5: and the muscles have to carry out anaerobic respiration (to release energy). P6: The glucose is converted into lactic acid, P7: with only a limited amount of energy being produced P8: An oxygen debt builds up in the body as shown in the graph P9: High levels of lactic acid in the muscles P10: cause them to ache. P11: After running, the athlete breathes more rapidly / deeply than normal for 20 minutes (shown in the graph) P12: There is a recovery period (from the 10th minute until the 20th minute) P13:when oxygen is paid back (during aerobic respiration) P14: About 1/6 lactic acid is oxidised to carbon dioxide, water and energy. MAXIMUM: 10 marks

10

20 marks

TOTAL

8(a)

Able to explain how the inheritance happen Answer : P1: The situation involved is monohybrid inheritance. P2: The genotype of blood group A can be IAIA /1A10 P3: while the genotype of blood group B can be I BI B or IBIO. P4: Blood group 0 has a genotype, I OI O (while the genotype of blood group AB is I AI B. P5: Alleles 1A and IB are codominant P6: IO allele is recessive. P7: Mr. Nick is heterozygous dominant/IAIO (for his blood group A) P8: while his wife is heterozygous dominant/ IBI0 (for blood group B) P9: Mr. Nick and his wife produce haploid gametes/sperm/ovum (as a r e s u l t o f m e i o s i s ) P10: Mr. Nick produces (gametes with) genotypes IA /IO P11: (while) his wife (will) produce (gametes with) genotypes 1A/ lO P12: The gamete (IO) of Mr. Nick fuses with his wife's gamete (10) P13: to produce a zygote with genotype IIo. P14: (Thus, they will) produce an offspring with blood group 0. 1 1 1 1 1 1 1 1 1 1 1 1 1 1

(b)

MAXIMUM: 10 marks

10

(i) Able to explain how DNA fingerprinting is carried out. Answer: P1: Tissue samples are taken from the scene of a crime and DNA is extracted. P2: An enzyme breaks down the DNA into fragments. P3: The DNA fragments are classified according to size. P4: An alkali is added to separate the double-stranded D N A into single strands. P 5 : Each single strand is laid on a nylon membrane and

1 1 1 1 1 1

radioactive matter is added to it. A banding pattern appears. P6: An X-ray film is produced and the positions of black bands are compared with the part of DNA treated with radioactive matter. MAXIMUM: 4 marks (ii) Able to state the advantages and disadvantages of DNA fingerprinting Sample answer: Advantages: P1: DNA fingerprinting is more accurate than common fingerprinting as no two people have the same DNA fingerprints. P2: DNA fingerprinting is more efficient than blood-type identification because many people have the s a m e blood type obtain a highly accurate result P4: DNA samples last longer than fingerprints. P5: Mixed DNA samples can still be used. P6: DNA evidence is harder to clean up compared to fingerprints. Disadvantages: P7: DNA samples may be degraded by adding chemicals, and this will affect the accuracy of the technique. P8: Human errors are possible when different procedures and standards are used in DNA fingerprinting. MAXIMUM: 6 marks TOTAL 20 marks 1 6 1 1 1 1 1 1 4

P3: DNA fingerprinting requires only a small amount of DNA to 1

9(a)

(i) Able to explain why most plants cannot colonise and grow in the swamps. Sample answer: P1: The ground is too soft and unable to support plants, P2: The water-logged / muddy swamps provide very little oxygen for root respiration. P3: The swamp water has a high concentration of salt and is hypertonic. P4: The plants growing in swamp will have the problem of dehydration. P5: Seeds that fall into the muddy swamp will die of dehydration / insufficiency of oxygen. P6: The swamp is exposed to strong sunlight and intense heat. P7: As a result, the plants growing there will lose water very fast by transpiration. MAXIMUM: 5 marks (ii) Able to explain how the mangrove trees adapt themselves to the harsh living conditions Sample answer: P1: Root system which is highly branched and spreads over a big area to give good support to the plants. P2: Pneumatophores (breathing roots) which grow protruding upwards above the ground. P3: The plant cells have high concentration of cell sap. P4: Hence, the cells are able to withstand the high salt content of the swamp. P5: Excess salt is eliminated through hydatodes found at the lower epidermis of leaves. P6: Viviparous seeds which germinate while still attached to the parent plant. P7: The long radical produced will let the seedling stick into the ground and not submerge or drift away. P8:Thick cuticle and sunken stomata which help to reduce the rate of transpiration. 1 1 1 1 1 1 1 1 1 1 1 5

MAXIMUM: 5 marks (b) Able to describe the effects of unplanned development and improper management of the ecosystem. P1: The leave canopy in the forest protects the soil from excess rain water. P2: When the forest is cleared, the soil is exposed to rain (water) / wind. P3: this will cause soil erosion P4:The soil that is exposed to wind will be blown to another area, P5: while soil that is exposed to rain water will be eroded and deposited at the bottom of the river / pond /lake. P6:The soil at the hill slopes can (also) be washed away by heavy rain water P7: resulting in land slides. P8: (The deposited soil will) cause the water level to increase rapidly when it rains and P9: this will in turn cause flash floods. P10:Wild life species will also be threatened P11: when their habitat is destroyed. P12: Global warming will occur P13:due to an increase in the Earth's temperature, P14:which is caused by excess emissions of carbon dioxide/ methane/ CFC /nitrogen dioxide (into the atmosphere). P15:These gases trap the heat that is reflected by the Earth. P16:The thinning of the ozone layer occurs P17: when the ozone layer (that protects the Earth from ultraviolet radiation) is destroyed by chlorofluorocarbons (CFC). MAXIMUM: 10 marks 1 1 1 1 1 1 1 1 1 1 1 1 1 1 1 1 1

10

Você também pode gostar